Test Bank – ACP: Neurology

The Dizzy Patient

1. A 50-year-old man presents to your clinic complaining of ringing in his right ear and of feeling as if the room is spinning around him. The latter symptom lasts for hours at a time. His wife points out that he is also losing his hearing. You suspect that he is suffering from Meniere syndrome.

Which of the following statements about Meniere syndrome is false?
A. Meniere syndrome is characterized by tinnitus, vertigo, and hearing loss
B. Patients suffering from Meniere syndrome can have sudden attacks, during which they fall to the ground
C. Episodes of vertigo and deafness in Meniere syndrome can last for hours to a day
D. The deafness associated with Meniere syndrome begins with an inability to hear low-frequency sounds and later progresses to involve higher frequency sounds

Key Concept/Objective: To be familiar with the presentation of Meniere syndrome

Although episodes of vertigo associated with Meniere syndrome usually last from hours to a day, deafness is constant after it develops. The deafness is characterized initially by a compromised ability to hear low-frequency sounds. Meniere syndrome is the result of excessive production and decreased absorption of inner ear endolymphatic fluid, resulting in endolymphatic hydrops. Patients usually complain of unilateral attacks of tinnitus (some have bilateral attacks), vertigo, and fullness in an ear. Deafness develops over time. Tumarkin otolithic crisis is a sudden spell of falling to the ground associated with Meniere syndrome. Meniere syndrome can be associated with nausea and vomiting. (Answer: C— Episodes of vertigo and deafness in Meniere syndrome can last for hours to a day)

2. A 52-year-old patient is diagnosed with Meniere syndrome. She has experienced several episodes of vertigo and now has right ear deafness. She has responded well to oral meclizine for the vertigo and to antiemetics for the associated vomiting. She wants to know if there are ways to avoid having similar attacks in the future, as this last one was particularly disabling.

Which of the following is not a long-term treatment for Meniere syndrome?
A. Salt restriction
B. The Epley maneuver
C. Diuretics
D. Weight loss

Key Concept/Objective: To know the treatment options for Meniere syndrome

Meniere syndrome is the result of increased levels of endolymphatic fluid. Treatments include salt restriction, diuretics, and weight loss. During attacks, vestibular suppressants and antiemetics can be useful for symptom relief. Refractory cases can be treated with surgical endolymphatic shunting, labyrinthectomy, and vestibular neurectomy. The Epley maneuver is intended to dislodge otoconia from the semicircular canal system and is therefore useful in the treatment of benign paroxysmal positioning vertigo (BPPV). (Answer: B—The Epley maneuver)

3. A 66-year-old man presents to the emergency department complaining of nausea, vomiting, dizziness, and unsteadiness on his feet. The onset of these symptoms was acute. He states he has had a few similar episodes during the past month; each episode lasted a few hours. His medical history is notable for hypertension, hyperlipidemia, and coronary artery disease. He continues to smoke cigarettes despite his difficulties with heart disease.

The treatment of this patient should include which of the following measures?
A. Performance of the Epley maneuver
B. Administration of vestibular suppressants
C. Reassurance and the prescription of a salt-restricted diet
D. A thorough neurologic examination and consideration of neurovascular imaging

Key Concept/Objective: To be able to recognize patients at risk for vertebrobasilar insufficiency and to understand the need to search for this disorder in such patients presenting with vertigo

Episodes of vertigo in middle-aged patients should prompt consideration of vertebrobasilar insufficiency as a cause. The vertigo can represent transient ischemic attacks, which are warning signs of possible vertebrobasilar occlusion, a life-threatening condition. One should have even more concern about this condition in those with known risk factors for vascular disease or those with established vascular disease. Diagnosis often requires magnetic resonance angiography, a conventional angiogram, or both. This patient’s episodes are very much a cause of concern, given his medical history and the recent onset of multiple attacks. Positive findings on neurologic examination would be even more compelling. (Answer: D—A thorough neurologic examination and consideration of neurovascular imaging)

4. A 30-year-old woman presents to the emergency department complaining of the room “moving all around her,” nausea, and one episode of emesis. She states she was well until the week before, when she had a cold. Nasal congestion, a nonproductive cough, and a runny nose characterized the latter. She has no previous medical problems. Examination is notable only for torsional-horizontal nystagmus and difficulty with balance. You suspect she has vestibular neuritis.

Which of the following statements regarding vestibular neuritis is false?
A. It is commonly associated with upper respiratory tract infection
B. It is rare
C. It usually lasts from days to weeks
D. It can be treated with vestibular suppressants during the first few days of the illness

Key Concept/Objective: To understand the presentation and clinical features of vestibular neuritis

Vestibular neuritis is quite common, second only to BPPV as a cause of vertigo in most dizziness clinics. It is frequently associated with an upper respiratory tract infection and usually lasts from days to weeks. In most patients, symptoms improve within 1 to 2 days, and resolution occurs within 6 weeks of onset. The characteristic physical finding is torsional and horizontal nystagmus, with the slow component being toward the affected side. During the early phase of the illness, vestibular suppressants such as meclizine and diazepam can be useful in treating symptoms of vertigo. However, these agents should not be used for prolonged periods, because they may hinder the development of central compensation. Vertigo should be considered in the diagnosis of patients presenting with symptoms and signs consistent with vestibular neuritis, especially in the setting of diabetes or hypertension. Hearing is uniformly preserved. (Answer: B—It is rare)

For more information, see Solomon S, Frohman EM: 11 Neurology: I The Dizzy Patient. ACP Medicine Online (www.acpmedicine.com). Dale DC, Federman DD, Eds. WebMD Inc., New York, March 2005

Diseases of the Peripheral Nervous System

5. A 38-year-old white man presents to clinic with a complaint of sharp, “lightning-like” pain in both feet.
He has had no recent trauma. His problem has been slowly progressing, and he is very concerned that this disorder will lead to disability. You suspect a peripheral neuropathy.

Which of the following statements regarding peripheral neuropathy is false?
A. A peripheral nerve disease can manifest as a dysfunction of motor, sensory, or autonomic systems
B. Nerve conduction studies can provide support for a diagnosis of neuropathy and are a relatively objective way to follow the course of the disease
C. Complete pain relief should be the goal when using medications to treat neuropathic pain
D. Drugs used to treat neuropathic pain include tricyclics, carbamazepine, and gabapentin

Key Concept/Objective: To understand the diagnosis and treatment of peripheral nervous system diseases

Peripheral nervous system disorders produce combinations of motor, sensory, and autonomic symptoms. These symptoms are primarily determined by the class of nerve fibers affected (e.g., motor or sensory fibers) and by the location of the lesions, rather than by the etiology of the process. Neurophysiologic testing (especially nerve conduction studies) can be considered a routine part of the evaluation of any patient with polyneuropathy. Nerve biopsy is best reserved for patients with disabling neurologic symptoms and signs. Nerve conduction studies can provide support for a diagnosis of neuropathy and are a relatively objective way to follow the course of the disease. These studies may also allow a clinician to infer whether a disease is affecting primarily axons or their myelin sheaths—an important distinction in the differential diagnosis of polyneuropathy. Needle electromyography is a complementary investigation that is usually performed at the same time as nerve conduction studies. In a patient with peripheral neuropathy, electromyography is helpful in detecting small degrees of axon loss that may go undetected by nerve conduction studies. The symptoms of neuropathy may be treatable even if the cause of the neuropathy is untreatable or unknown. With simple measures, many patients can have meaningful relief of symptoms. Medications can be useful in the management of neuropathic pain, but the goals of therapy should be realistic. Complete pain relief is unlikely. Therefore, the aim of therapy should be to make the pain more tolerable without adding intolerable side effects of medication. Of the many drugs that can be tried for neuropathic pain, tricyclics (especially amitriptyline) and carbamazepine are still most frequently used, although gabapentin is increasingly used as a first-line agent. (Answer: C—Complete pain relief should be the goal when using medications to treat neuropathic pain)

6. A 32-year-old woman presents to the emergency department with a complaint of weakness. Yesterday, she noticed some prickling paresthesias in her feet. Today, when she awoke, she noticed weakness in both legs; this weakness has rapidly worsened. You admit her to the hospital with a presumptive diagnosis of Guillain-Barré syndrome (GBS).

Which of the following statements regarding GBS is true?
A. Another name for GBS is chronic inflammatory demyelinating polyneuropathy
B. The fundamental pathologic event in GBS is the stripping of myelin from axons by macrophages, which occurs in a patchy fashion throughout the peripheral nervous system
C. Several studies have proved that there is a link between a preceding Shigella dysentery infection and GBS
D. A cardinal feature of GBS is the asymmetrical pattern of involvement

Key Concept/Objective: To understand the pathophysiology and clinical presentation of GBS

GBS, or acute inflammatory demyelinating polyradiculoneuropathy, is the most common cause of acute generalized paralysis in the Western world. Chronic inflammatory demyelinating polyradiculoneuropathy is an immune-mediated neuropathy whose onset is insidious, with symptoms and signs developing over weeks to months. In contrast, the initial course of GBS is rapid. Most often, the first symptom of GBS is prickling paresthesia, beginning in the feet and spreading proximally hour by hour. Weakness is noticed some hours to a few days later. Some patients have only motor symptoms without sensory symptoms. Classically, symptoms begin symmetrically in the distal limbs and proceed proximally (socalled ascending paralysis). Nerve conduction studies provide evidence of a demyelinating process affecting spinal roots and peripheral nerves (a demyelinating polyradiculoneuropathy). The fundamental pathologic event in GBS is the stripping of myelin from axons by macrophages, which occurs in a patchy fashion throughout the peripheral nervous system. A cascade of events involving cell-mediated and humoral immune mechanisms is assumed to be activated, and lymphocytic inflammatory infiltrates are often found in nerves and nerve roots by biopsy or at autopsy. Studies of the pathogenesis of GBS have focused on the potential roles of antecedent Campylobacter jejuni infection and the production of antiganglioside autoantibodies, both of which occur in a large number of patients with GBS. (Answer: B—The fundamental pathologic event in GBS is the stripping of myelin from axons by macrophages, which occurs in a patchy fashion throughout the peripheral nervous system)

7. A 68-year-old African-American patient with type 2 diabetes mellitus presents to clinic for a 6-month follow-up visit. His daily capillary blood glucose level has been ranging from 160 to 190 mg/dl, and he has not been adhering to his diet. His major complaint today is a burning pain in both feet. On foot examination, you discover a 1 cm ulcer on the plantar surface of the left foot and a loss of light touch sensation in both feet. You make a diagnosis of diabetic polyneuropathy.

Which of the following statements regarding diabetic polyneuropathy is false?

A. There is a strong correlation between the presence of diabetic polyneuropathy, retinopathy, and nephropathy
B. Autonomic diabetic neuropathy can occur and cause orthostatic hypotension, impaired gastrointestinal motility, or blunting of the sympathetic response to hypoglycemia
C. The classic distribution for diabetic polyneuropathy is the glove-andstocking distribution
D. The severity of the polyneuropathy correlates more closely with the duration of diabetes than the degree of hyperglycemia (mean glycosylated hemoglobin)

Key Concept/Objective: To understand the clinical presentation of diabetic polyneuropathy

Peripheral neuropathy is common in patients with diabetes mellitus. Of the various types of diabetic neuropathy, by far the most common is a distal, symmetrical sensorimotor neuropathy, commonly referred to as diabetic polyneuropathy. In one prospective, population-based study of Americans of mainly northern European ancestry, diabetic polyneuropathy was found in 54% of patients with type 1 diabetes mellitus and in 45% of patients with type 2 diabetes mellitus. However, symptomatic polyneuropathy occurred in only 15% of the cohort, and none of the patients had disabling neurologic deficits. The severity of the polyneuropathy correlated more closely with the degree of hyperglycemia (mean glycosylated hemoglobin) than with the duration of diabetes. In this study and other large studies, the prevalence of diabetic polyneuropathy increased with the duration of diabetes, and a strong correlation existed between the presence of diabetic polyneuropathy, retinopathy, and nephropathy. An important practical corollary of these observations is that a diagnosis of diabetic polyneuropathy in a patient with newly diagnosed diabetes but without other diabetic complications is likely to be incorrect. Diabetic polyneuropathy has the classic so-called glove-and-stocking distribution of symptoms, usually a combination of sensory loss and an unpleasant feeling of numbness or burning. Sensory loss in the feet and fingers and mild weakness in the feet and ankles are typical. Diabetic polyneuropathy can be expected to worsen slowly over years. The other varieties of diabetic neuropathy usually occur on a background of diabetic polyneuropathy. Some degree of diabetic autonomic neuropathy is found in most patients with diabetic polyneuropathy, although in some patients, the autonomic symptoms and signs predominate. Orthostatic hypotension, impaired gastrointestinal motility (including gastroparesis), and blunting of the sympathetically mediated warning symptoms of hypoglycemia are important management problems. (Answer: D—The severity of the polyneuropathy correlates more closely with the duration of diabetes than the degree of hyperglycemia [mean glycosylated hemoglobin])

8. A 49-year-old man presents to the emergency department with abrupt onset of right facial weakness. He experienced a respiratory infection 2 weeks ago and has had a dull ache behind the right ear for 2 days. This morning while shaving, he noticed a drooping of the right side of his face. Neurologic examination reveals a neuropathy of cranial nerve VII, with complete paralysis of the right upper face and forehead. Hearing, taste, and sensation are normal, and the other cranial nerves are functioning normally. No rash or shingles lesions are noted.

Which of the following clinical features seen in this patient suggests a poorer prognosis and would prompt more aggressive medical treatment (e.g., prednisone and acyclovir)?
A. Age younger than 60 years
B. Normal hearing
C. Normal taste
D. Complete paralysis
E. Abrupt onset of symptoms

Key Concept/Objective: To be able to recognize and manage acute Bell palsy and to know the features that are associated with a poor prognosis and that suggest the need for early medical therapy This patient has acute, idiopathic, facial neuropathy (Bell palsy). The abrupt onset of unilateral facial weakness with lower motor neuron (forehead) involvement preceded by pain behind the ear is classic. Over 80% of patients eventually recover fully. A course of corticosteroids and acyclovir may hasten recovery and is appropriate for patients with clinical features portending a poorer prognosis. Such features include severe (complete) paralysis, older age, hyperacusis, altered taste, and electromyographic evidence of axonal degeneration. Recent evidence implicates HSV type 1 infection in many patients, and trials of facial nerve decompression have not demonstrated efficacy. (Answer: D—Complete paralysis)

9. A 68-year-old diabetic man presents to the office for evaluation of double vision and right retro-orbital headache. Neurologic examination reveals right ptosis. The eye is deviated laterally and inferiorly. The right pupil is not dilated and reacts normally to light and accommodation. The patient has decreased sensation in his feet to pin and light touch, but the rest of the examination results are normal.

Which of the following is indicated for this patient at this time?
A. Cranial CT
B. Magnetic resonance angiography
C. Cerebral angiography
D. Neurosurgical consultation
E. Reassurance and no further evaluation at this time

Key Concept/Objective: To be able to recognize oculomotor (third cranial nerve) paralysis and to understand the causes and implications of pupil-sparing third nerve palsy

This diabetic patient has developed an isolated third-nerve palsy. The presenting symptoms are fairly typical: the sudden development of ptosis with a “down-and-out eye” and a retro-orbital headache. The major consideration in the differential diagnosis is a mass lesion along the course of the third cranial nerve, particularly an aneurysm in the circle of Willis. The absence of pupillary paralysis (pupil-sparing third-nerve palsy) seen in elderly, hypertensive, and especially diabetic patients points to a putative microvascular lesion of the third cranial nerve. The finding of pupillary sparing can be distinguished at the bedside and can obviate the need for expensive neuroradiologic imaging in most cases, though such investigation is still advised in young patients who are without hypertension or diabetes. (Answer: E—Reassurance and no further evaluation at this time)

10. A 52-year-old man presented to the urgent care center 2 weeks ago with severe left shoulder pain. He was diagnosed with bursitis and treated with NSAIDs. The pain has gradually improved, but the patient has scheduled an office visit because he is concerned about weakness of the left arm. On examination, the patient has full passive range of motion of the arm and shoulder without pain. Marked atrophy and weakness are noted in the left deltoid and shoulder girdle muscles. Biceps and triceps reflexes are absent. The remainder of the examination is unremarkable.

Which of the following is the most likely diagnosis for this patient?
A. Thoracic outlet syndrome
B. Brachial plexitis (Parsonage-Turner syndrome)
C. Rotator cuff tear
D. Spinal cord tumor
E. Lacunar infarction

Key Concept/Objective: To be able to recognize the syndrome of brachial plexus neuropathy (Parsonage-Turner syndrome)

Patients with Parsonage-Turner syndrome present with severe shoulder pain (frequently confused with bursitis) evolving to weakness of the muscles of the shoulder girdle, arm, or hand. This uncommon condition, also known as brachial plexitis or neuralgic amyotrophy (amyotrophy refers to wasting of muscle after denervation), appears to be an inflammatory peripheral neuropathy with causalgic pain. Affected patients are usually young or middle-aged men. It may also be seen in patients who are recovering from serious illness. Gradual, spontaneous recovery generally occurs; no specific therapy has been shown to be efficacious. Other less common causes of brachial plexopathy are irradiation (usually involving the upper plexus, cervical roots 5 through 7), trauma (as might be caused by the arm being jerked upwards or downwards), and malignant infiltration (usually involving cervical root 8 through thoracic root 1). Lumbosacral plexopathies are less common and are usually caused by diabetic neuropathy, malignant infiltration, or retroperitoneal hemorrhage. (Answer: B—Brachial plexitis [Parsonage-Turner syndrome])

11. A 72-year-old man has experienced numbness of his hands and feet for at least 5 years. Several times recently, he has tripped over the right foot. There is no history of diabetes, alcoholism, or other medical illness. He takes no medications or vitamin preparations. Examination reveals reduced pinprick and light-touch sensation in a stocking-and-glove distribution, which is worse in the feet than the hands. He has bilateral pes cavus and bilateral footdrop, which is worse on the right side. Chest x-ray, hematology group, chemistry panel, sedimentation rate, hemoglobin A1c level, antinuclear antibody assay, and protein electrophoresis results are all normal or negative. Electromyography (EMG) shows primarily axonal degeneration of motor and sensory nerves of the lower and upper extremities.

Which of the following is most likely to identify the cause of this patient’s neuropathy?
A. Family history and examination of suspect family members
B. Screening of the urine for heavy metal
C. Nerve biopsy
D. Assessment of vitamin B12 levels
E. Liver function tests, including GGT

Key Concept/Objective: To understand the causes and evaluation of symmetrical polyneuropathy

Middle-aged or older patients with mild, nondisabling, slowly progressive polyneuropathy frequently have no identifiable etiology on routine examination and laboratory testing. In the absence of diabetes, alcohol abuse, medications, or other systemic illness, more extensive studies (e.g., screening of urine for heavy metals, fat aspiration for amyloid, sural nerve biopsy) rarely reveal an etiology. Many of these patients in fact suffer from inherited polyneuropathy. They frequently are asymptomatic for years, then develop numbness (prickling or pins-and-needles sensation suggests an acquired cause). The routine family history usually is negative. However, specific questioning about relatives with foot deformities or who need special shoes, braces, or gait-assist devices may be revealing. Neurologic evaluation of first-degree relatives who might be affected is more cost-effective than additional laboratory tests. (Answer: A—Family history and examination of suspect family members)

12. A 34-year-old woman presents to the emergency department complaining of a “pins-and-needles” sensation in her feet and legs, which began yesterday. Initially, this sensation involved only her feet, but it has gradually moved up to involve her ankles, calves, and thighs. She has had difficulty walking for the past few hours. She recalls a recent episode of bloody diarrhea, but otherwise she has been in good health. On examination, there is weakness of the foot and leg muscles in a symmetrical distribution. Deep tendon reflexes are absent at the knees and ankles. Plantar reflex is flexor. Fasciculations are not seen. Inconsistent results are obtained on sensory exam. Cranial nerves are intact. The rest of the physical exam is normal. CSF examination is normal except for an elevated protein level.

Which of the following is the most likely diagnosis for this patient?
A. Botulism
B. Motor neuron disease
C. GBS
D. Poliomyelitis
E. Acute HIV infection

Key Concept: To understand the diagnosis of Guillain-Barré syndrome

Inflammatory demyelinating polyradiculoneuropathy is the most common variant of the GBS. The disease incidence is approximately 1 case per million population per month. Antecedent infections with viruses, mycoplasmas, or Campylobacter jejuni occur in one half to two thirds of patients. Patients typically present with paresthesias in the feet, which progress proximally. Pain is common early in the course of illness. An areflexic motor paralysis and an acellular increase in total protein levels in the CSF develop in most patients within 1 week. Paralysis can progress rapidly; early diagnosis facilitates early hospitalization for appropriate nursing and medical care. Up to one third of patients require ventilatory support. Complete or near-complete recovery is the rule in 85% of patients. Early in the course of disease, many patients are misdiagnosed as having anxiety disorder, as malingering, or as having other psychiatric illness. Careful neurologic examination and early neurologic consultation in suspect cases are advised. Early cranial nerve involvement would suggest botulism, myasthenia gravis, or the Miller-Fisher variant of GBS. Muscle cramps, normal sensation, fasciculations, and preserved or hyperactive deep tendon reflexes characterize motor neuron disease (e.g., ALS). Poliomyelitis was the major consideration in the differential diagnosis before the development of the polio vaccine, but it is now rare. A GBSlike syndrome with CSF pleocytosis can occur early in acute HIV infection. (Answer: C—GBS)

For more information, see Chalk CH, Dyck PJ: 11 Neurology: II Diseases of the Peripheral Nervous System. ACP Medicine Online (www.acpmedicine.com). Dale DC, Federman DD, Eds. WebMD Inc., New York, August 2004

Diseases of Muscle and the Neuromuscular Junction

13. A 46-year-old man presents with difficulties of gait and weakness of the face, neck, and hands. On examination, the patient has a “hatchet-face” appearance, with obvious wasting of the temporalis and masseter muscles. Bilateral eyelid ptosis without extraocular weakness is noted. There is prominent neck flexion but not extension, weakness, and atrophy. Interosseous atrophy and a bilateral footdrop are noted. The patient has prominent frontal baldness and testicular atrophy. When asked to grip the examiner’s hand, the patient has difficulty relaxing the grip quickly. Percussion of the thenar eminence leads to slow relaxation. The serum creatine phosphokinse (CPK) level is normal.

Which of the following is the most likely diagnosis for this patient?
A. Becker muscular dystrophy
B. Duchenne muscular dystrophy
C. Autosomal recessive sarcoglycanopathy
D. Autosomal dominant fascioscapulohumeral dystrophy
E. Myotonic dystrophy (MD)

Key Concept/Objective: To know the clinical signs and symptoms of MD

This patient likely has MD, the most common muscular dystrophy in adults. MD is an autosomal dominant disorder; patients present with a unique constellation of clinical features: ptosis, temporal and masseter atrophy, atrophy of the sternocleidomastoid muscles (with sparing of other posterior neck muscles), atrophy of distal musculature, dysarthria, and dysphagia. Myotonia (the inability to quickly relax a firm hand grip) should be specifically sought; its presence is characteristic. Most patients have disorders of the cardiac conduction system. Other features include frontal baldness, testicular atrophy, cataracts, mild mental dysfunction, GI motility disorders, and hypersomnia. The CPK level is normal or only mildly increased. EMG is diagnostic, revealing myotonic discharges. (Answer: E— Myotonic dystrophy [MD])

14. A 60-year-old woman is referred to you for a preanesthetic medical evaluation before elective total knee arthroplasty. Her degenerative joint disease of the knees has led to severe pain at rest and while walking, and conservative treatment measures have failed. Otherwise, she has been in good health and takes no medications. The patient has been told that no one in her family should undergo anesthesia with halothane, because her father developed a high fever and died during a cholecystectomy.

What is the most appropriate step to take next in the care of this patient?
A. Proceed with surgery; no additional precautions are needed
B. Proceed with surgery but alert the anesthesiologist to avoid halothane anesthetics
C. Proceed with surgery; administer I.V. dantrolene if fever develops
D. Cancel surgery and refer for caffeine-halothane contraction test
E. Cancel surgery and advise against any elective surgical procedure

Key Concept/Objective: To be able to recognize malignant hyperthermia and to understand the most appropriate screening measures for patients suspected of having this disease

Malignant hyperthermia is an autosomal dominant disorder caused by a defect on chromosome 19q13, leading to a mutation of the ryanodine receptor (RyR) gene. Mutations of RyR cause accelerated calcium release from the sarcoplasmic reticulum during general anesthesia with compounds such as halothane, ether, and succinylcholine. This leads to a rapid increase in metabolism, dramatic elevations of body temperature, acidosis, muscle rigidity, myoglobinuria, and death. A careful family history may give clues to the diagnosis and should prompt referral for a muscle biopsy and in vitro caffeine-halothane contraction testing. Patients with malignant hyperthermia can usually safely undergo anesthesia with nitrous oxide, thiopental, and nonpolarizing muscle relaxants. I.V. dantrolene is effective if administered early in the disease course, but patients who develop the syndrome still have a 7% mortality. (Answer: D—Cancel surgery and refer for caffeine-halothane contraction test)

15. A 24-year-old Asian man presents to the emergency department with an attack of profound weakness after a meal with friends. He reports that for several years he has had similar episodes after exercise and large meals.

Which of the following diagnostic tests should be performed immediately for this patient?
A. Testing for acetylcholine receptor antibodies
B. Assessment of serum potassium level
C. Assessment of serum thyroxine level
D. EMG
E. Assessment of urinary aldosterone level

Key Concept/Objective: To know the diagnosis of periodic paralysis

Both hyperkalemic and hypokalemic periodic paralysis are characterized by an abnormal serum potassium level at the time of symptom occurrence. However, the potassium levels can be normal between attacks, and thus, measurement of serum potassium during the period in which symptoms occur is the most important step to take next in treating this patient. Hyperkalemic periodic paralysis is caused by a defect of the sodium channel, precipitated by rest following exercise, stress, potassium administration, and the ingesting of certain foods. Hypokalemic periodic paralysis is caused by a defect in the calcium channel and is precipitated by the partaking of meals high in carbohydrates, rest following exercise, and excitement. If the potassium level is found to be low during attacks, secondary causes of hypokalemia (diuretics, hyperaldosteronism, laxatives, etc.) or thyrotoxicosis (especially in patients of Asian descent) should be sought. A serum potassium level that is elevated without apparent cause is suggestive of hyperkalemic periodic paralysis. (Answer: B—Assessment of serum potassium level)

16. A 25-year-old woman presents for evaluation of progressive muscle weakness and fatigability for the past
9 months. She has otherwise been healthy and takes no medication except oral contraceptives. The weakness involves her face, neck, and arms. The weakness is worse toward the end of the day and after repetitive activity. Her symptoms improve somewhat with sleep or rest. On examination, bilateral ptosis and extraocular muscle weakness are noted, particularly on upward and lateral gaze. Attempts at forced smiling produce a snarling expression. She has prominent neck muscle weakness. Her speech has a nasal quality. Moderate weakness is evident on upper-extremity muscle testing. Deep tendon reflexes and plantar reflexes are normal. Routine laboratory examinations, including CBC, chemistry panel, and thyroid function testing, are normal.

What is the most likely cause of this patient’s symptoms?
A. Motor neuron disease
B. Lambert-Eaton myasthenic syndrome
C. Myasthenia gravis
D. Congenital myasthenia
E. Polymyositis

Key Concept/Objective: To be able to diagnose myasthenia gravis and to distinguish it from other myasthenic and myopathic syndromes

Primary care physicians should suspect myasthenia gravis in patients who have progressive skeletal muscle weakness and fatigability. The illness typically presents in young women or older men as weakness of the eyelids and extraocular muscles, which leads to ptosis and diplopia. Patients develop weakness of the neck extensors and bulbar weakness that leads to dysarthria and dysphagia. Proximal weakness may present as progressive weakness experienced when climbing stairs or rising from a chair. Some patients complain of weakness combing their hair. Fluctuation of symptoms and fatigue with activity are characteristic. Deep tendon reflexes and the plantar reflex are normal. The presence of antibodies against the acetylcholine receptor and a positive EMG are diagnostic. LambertEaton myasthenic syndrome is frequently associated with small cell lung cancer; its symptoms are ptosis, diplopia, fatigability, and muscle weakness. Features distinguishing it from myasthenia gravis include hyporeflexia, autonomic dysfunction, and an increase in muscle strength after several seconds of maximal effort. Congenital myasthenia presents in infancy, childhood, or, occasionally, young adulthood. Motor neuron disease can present as muscle aches, weakness, and fatigue. The first manifestation may be asymmetrical distal weakness, with progressive wasting and atrophy of muscles or difficulty with chewing, swallowing, and moving the face and tongue. Fasciculation, caused by spontaneous twitching of motor units, is characteristic. With prominent corticospinal involvement, hyperactivity of the deep tendon reflexes is found. (Answer: C—Myasthenia gravis)

For more information, see Dalakas MC: 11 Neurology: III Diseases of Muscle and the Neuromuscular Junction. ACP Medicine Online (www.acpmedicine.com). Dale DC, Federman DD, Eds. WebMD Inc., New York, March 2005

Cerebrovascular Disorders

17. A 72-year-old man presents to the emergency department for evaluation. He is accompanied by his wife, who provides a history of his present illness. The patient was in his usual state of health until 1 hour ago, when he lost the use of his right arm and leg after sliding out of his chair. He is being treated for hypertension, diabetes, and dyslipidemia, all of which have been under moderately good control for many years. On physical examination, the patient has a dense paresis of his right upper and lower extremities. His language is unintelligible, and he is near mute. Intravenous fluids are started, serum is collected, and he is urgently transferred to radiology.

Which of the following statements regarding diagnosis of and therapy for acute ischemic stroke is true?
A. The brain should first be imaged with noncontrast CT
B. Anticoagulation should be the initial medical therapy for most ischemic stroke patients
C. The use of recombinant tissue plasminogen activator (rt-PA) for ischemic stroke should occur within 6 hours of symptom onset
D. Heparin should not be used for prophylaxis of deep vein thrombosis in patients with ischemic stroke

Key Concept/Objective: To understand the treatment of acute ischemic stroke

Noncontrast CT reliably distinguishes acute intracerebral hemorrhage from ischemia. This distinction is critical because the management of hemorrhagic stroke is substantially different from that of ischemic stroke. Anticoagulation is commonly used in the acute setting to prevent progressive or recurrent thromboembolic events. Nevertheless, the efficacy and safety of anticoagulation for this purpose are not well established, and its role in clinical stroke management is controversial. Most patients with ischemic stroke should not be treated with anticoagulation. Aspirin is recommended as initial therapy for most acute stroke patients. However, aspirin should be withheld for at least 24 hours after administration of thrombolytics. Although the FDA has approved the use of intravenous rt-PA for acute ischemic stroke up to 3 hours after the onset of symptoms, physicians should strive to treat patients as quickly as possible, because earlier therapy is associated with better outcomes. Prophylaxis for deep vein thrombosis should be instituted early with heparin. For patients in whom heparin is contraindicated (e.g., patients with acute hemorrhage), pneumatic compression stockings are employed. (Answer: A—The brain should first be imaged with noncontrast CT)

18. A 44-year-old black man is brought by ambulance to the emergency department for evaluation. He was found earlier by his wife, who states that he was in his usual state of health when he went to bed the previous day. This morning, the patient awoke with severe headache; he was excessively groggy, and his speech was slurred. His wife called an ambulance. His only known medical problem is hypertension, for which he takes a thiazide diuretic. On physical examination, the patient’s blood pressure is 210/140 mm Hg. He is arousable to pain and loud voice only, and he is combative and confused. The patient is treated with topical nitroglycerin. Serum is collected, and he is urgently transferred to radiology.

Which of the following statements regarding hemorrhagic stroke is true?
A. Blood pressure should be normalized rapidly in patients with intracranial hemorrhage
B. Though hypertension is a clear risk factor for intracerebral hemorrhage
(ICH), it is not known to increase the risk of aneurysmal rupture
C. In ICH, the volume of hemorrhage and the level of consciousness are the two most powerful predictors of outcome
D. Beta blockers have been shown to improve outcome in subarachnoid hemorrhage

Key Concept/Objective: To understand the therapy for hemorrhagic stroke

ICH volume and consciousness level are the two most powerful predictors of outcome in ICH. Observations suggest that about one third of ICHs expand in the first 24 hours. Some investigators have juxtaposed this fact with a need to lower blood pressure in acute ICH. No trial has demonstrated that this action is necessary. The American Heart Association guidelines recommend only that mean arterial blood pressure be kept lower than 130 mm Hg in patients with a history of hypertension. Hypertension and cigarette smoking are clear risk factors for aneurysmal rupture. A family history of subarachnoid hemorrhage in first-degree relatives is also a risk factor for aneurysm rupture (about 4%), but routine screening is not recommended. Because it has been shown to improve outcome, nimodipine, a calcium channel blocker, is begun on the first day and continued for 21 days. (Answer: C—In ICH, the volume of hemorrhage and the level of consciousness are the two most powerful predictors of outcome)

19. A 73-year-old woman comes to the emergency department with a sudden onset of confusion, as described by her family. The patient was well and in her usual state of health until this event. She had no loss of consciousness or focal neurologic findings. She is independent in her activities of daily living. Her medical history is unremarkable except for a remote history of hypertension, and she is currently taking no medications. Her examination reveals a blood pressure of 140/84 mm Hg, a heart rate of 108 beats/min, and a temperature of 98.6° F (37° C). Her neck reveals no bruits, and her heart examination reveals an irregular rhythm without murmurs or extra sounds. Neurologic examination shows her to be alert and in no acute distress. Cranial nerve, sensory, motor, cerebellar, and reflex examinations are normal. Mental status examination reveals an intact ability to follow commands and verbal fluency but difficulty in associating meaning with words.

Which of the following is the most likely explanation of this patient’s symptoms?
A. Acute ischemia to the left temporal lobe
B. Acute infectious process
C. Acute ischemia to the brain stem
D. Central nervous system neoplasia
E. Acute lacunar infarct of the basal ganglia

Key Concept/Objective: To understand the pathogenesis of acute expressive aphasia and that expressive aphasia in the absence of other neurologic findings may be mistaken for confusion

This patient presents with what the family described as an acute confusional episode, but on examination by the medical team, she was noted to be able to follow commands. Her past history is essentially unremarkable, and she is on no medications and has no convincing evidence of a systemic or local infectious process. The major findings on her clinical examination are probable atrial fibrillation and expressive aphasia in the absence of other neurologic symptoms. This patient has almost assuredly experienced a thromboembolism to the speech area of the left temporal lobe, caused by her atrial fibrillation. (Answer: A—Acute ischemia to the left temporal lobe)

20. An 85-year-old man presents to the emergency department after being found with a diminished level of consciousness and dense left facial and left upper extremity paresis. Examination reveals a blood pressure of 162/90 mm Hg and a normal heart rate and rhythm. His physical examination confirms the above findings. Carotid Doppler examination reveals minimal atherosclerotic narrowing. An echocardiogram reveals minimal left ventricular hypertrophy and no intra-atrial or intraventricular thrombus. Serologic studies reveal a normal sedimentation rate of 32 mm/hr, a positive rapid plasma reagin (RPR) agglutination test at a titer of 1:8, and a positive fluorescent treponemal antibody (FTA) test.

What is the best step to take next in the management of this patient?
A. Begin high-dose intravenous penicillin therapy
B. Obtain cerebrospinal fluid for VDRL (Venereal Disease Research
Laboratories) test
C. Obtain a CT scan of the head
D. Begin aspirin therapy
E. Begin antihypertensive therapy

Key Concept/Objective: To understand that an imaging study of the brain is necessary in the acute phase of a cerebrovascular accident (CVA) to differentiate an intracerebral bleed from thrombosis

This patient was found to have a dense hemifacial and left upper extremity hemiparesis. His examination and ancillary studies failed to reveal an obvious arterial or cardiac source of an embolism. His elevated blood pressure is a normal finding in the acute phases of a CVA and generally should not be specifically treated unless (1) the patient is a candidate for thrombolytic therapy; (2) it is after hemorrhagic conversion of the infarct; (3) the patient has an aortic dissection; or (4) the patient has hypertensive encephalopathy. This patient had a positive RPR and FTA, which suggests a prior syphilitic infection and raises the possibility that his CVA was caused by meningovascular syphilis. However, before treatment is begun or the diagnosis is confirmed with a lumbar puncture, a head imaging study is necessary to exclude hemorrhage as the etiology of his symptoms. (Answer: C— Obtain a CT scan of the head)

21. A 32-year-old woman presents with a sudden onset of right-sided hemiparesis and headache. She has no history of cardiovascular or neurologic disease and was well before the onset of her symptoms. She uses no tobacco, alcohol, or illicit drugs. Her examination reveals a normal blood pressure and pulse. Her neck examination reveals no bruits. Her heart and lung examinations are normal, and her neurologic examination confirms the above findings. Laboratory studies reveal a sedimentation rate of 20 mm/hr, a normal comprehensive profile, and a negative urinary drug screen. Antinuclear antibody test was positive at 1:40 dilution, and her anti-dsDNA (double-stranded DNA) was negative. Imaging studies of her brain reveal acute ischemic transformation of her left temporal-parietal region.

What is the best step to take next in the management of this patient?
A. Biopsy of the temporal artery
B. Visual evoked potentials
C. Spinal fluid examination for oligoclonal bands
D. Carotid duplex examination
E. Administration of high-dose intravenous corticosteroids

Key Concept/Objective: To know that carotid dissection is a cause of acute CVA in young adults

This is a case of a CVA occurring in a young, otherwise healthy female. In such cases, illicit drug use should be considered as a potential contributing factor, but in this case, it is essentially ruled out by the lack of confirmatory history or urinary drug screen. A vasculitis, perhaps secondary to a systemic process such as SLE, should be considered. This patient’s sedimentation rate is normal, and her ANA is nonspecific and low. However, the absence of any illness preceding the onset of her symptoms decreases the probability of a systemic inflammatory process. Visual evoked potentials and CSF analysis would be recommended if multiple sclerosis were a serious possibility. This patient has a single CNS lesion in the distribution of a major cerebral vessel (middle cerebral artery) and no other findings on her imaging study that suggest multiple independent CNS lesions. In young adults, carotid artery dissection needs to be considered in the differential diagnosis of CVA. Dissection may be diagnosed noninvasively with ultrasonography. (Answer: D—Carotid duplex examination)

22. A 49-year-old man presents to the emergency department with acute onset of severe headache, photophobia, and decreased level of consciousness. His mother had a subarachnoid hemorrhage at 54 years of age. He has no personal history of hypertension, vascular disease, or elevated cholesterol levels. His examination reveals a blood pressure of 148/84 mm Hg and mild nuchal rigidity. A CT scan of the head fails to reveal an abnormality.

What is the best step to take next in the management of this patient?
A. Cerebral angiography
B. Beginning oral nimodipine
C. Lumbar puncture
D. Electroencephalogram
E. Carotid Doppler examination

Key Concept/Objective: To understand that CT scanning of the brain is not 100% sensitive in excluding subarachnoid hemorrhage (SAH)

This patient presents with many of the classic findings of acute subarachnoid hemorrhage, including the sudden onset of a severe headache, diminished level of consciousness, and nuchal rigidity. Family history of aneurysm is present in about 4% of patients with SAH. Establishing the diagnosis early is necessary to improve long-term morbidity and mortality. It is important to note that CT scanning of the head is not 100% sensitive in excluding this “high-stakes” entity. In the presence of clinical suspicion and a negative imaging study, a lumbar puncture is necessary to look for the presence of xanthochromia and RBCs. (Answer: C—Lumbar puncture)

23. A 55-year-old man presents to the emergency department with sudden onset of tachycardia and lightheadedness. He has had no previous episodes of similar symptoms. He has a history of hypertension controlled with amlodipine. He is otherwise healthy. His examination reveals a blood pressure of 132/82 mm Hg and an irregular heart rate of 120 beats/min. His lung examination is normal, and his cardiac examination reveals an irregular rhythm, with no obvious murmur or extra sounds and S1 having variable intensity. An ECG reveals atrial fibrillation and left axis deviation. The chest x-ray is normal. An echocardiogram reveals normal left ventricular systolic and diastolic function and no thrombus or valvular abnormalities. TSH is 3.3.

Which of the following drugs would you give this patient to minimize the long-term risk of thromboembolism?
A. Intravenous heparin
B. Oral warfarin
C. Aspirin
D. Ticlopidine
E. Low-molecular-weight heparin

Key Concept/Objective: To understand that patients younger than 65 years who are without risk factors are at low risk for thromboembolism from atrial fibrillation

Risk factors such as hypertension, diabetes, previous CVA/TIA, and poor LV function, along with older age (> 65 years), are associated with a yearly risk of thromboembolism from atrial fibrillation of approximately 5%. This risk can be decreased to approximately 1% with warfarin and 2% to 3% with aspirin. The risk of thromboembolism is 1% without therapy in patients without risk factors and younger than 65 years. This patient has a history of controlled hypertension and has a normal echocardiogram, which decreases the probability that his hypertension has caused end-organ complications. He has no other risk factors for thromboembolism caused by his atrial fibrillation and likely has “lone atrial fibrillation.” In such patients, warfarin is not necessary, and aspirin may be a logical alternative in the absence of contraindications. Ticlopidine or newer antiplatelet agents may be of benefit when aspirin has failed. In addition, converting patients to sinus rhythm: either with electrical cardioversion or chemically: is a desirable outcome in such situations. Finally, agents to control ventricular rate (beta blockers, diltiazem, or digoxin) should be considered in this patient. (Answer: C—Aspirin)

For more information, see Morgenstern LB, Kasner SE: 11 Neurology: IV Cerebrovascular Disorders. ACP Medicine Online (www.acpmedicine.com). Dale DC, Federman DD, Eds. WebMD Inc., New York, October 2004

Traumatic Brain Injury

24. A 24-year-old man is brought to the emergency department by the emergency medical service (EMS). He suffered head trauma 20 minutes ago while playing football. Immediately after the event, he lost consciousness for 3 minutes and then woke up mildly confused. He complains of a moderate frontal headache. On physical examination, the patient’s vital signs are stable, his Glasgow Coma Scale (GCS) score is 15, and he has no focal signs on neurologic examination.

What interventions would be appropriate in the treatment of this patient?
A. Continue with observation and repeated neurologic examinations; repeat assessment with the GCS periodically; and consider imaging with a CT scan to rule out contusions
B. Continue with observation and repeated neurologic examinations;
repeat assessment with the GCS periodically; and obtain an MRI
C. Admit the patient for prolonged observation; obtain a CT scan to rule out contusions; and start I.V. mannitol for brain edema
D. Admit the patient to the ICU; obtain an MRI; and consider intraventricular monitoring of intracranial pressure (ICP)

Key Concept/Objective: To understand the appropriate treatment of mild traumatic brain injury (MTBI)

With an incidence of 180 per 100,000 people, MTBI is more common than any other neurologic diagnosis except migraine. MTBI is defined as any traumatic brain injury/concussion with loss of consciousness of 0 to 30 minutes, a GCS score of 13 to 15 on admission, posttraumatic amnesia or confusion lasting less than 24 hours, and no evidence of contusion or hematoma on CT. Although the emergency department evaluation and management of MTBI is controversial, the principal concern is with identifying evolving surgical lesions such as hematomas and contusions. In addition to history and examination, CT has become the mainstay of evaluation. Prolonged or deteriorating mental status or the presence of neurologic signs or other risk factors are still indications for CT scanning, observation, or both after MTBI. MRI promises to be very useful in the long-term management of moderate and severe TBI, as well as in the documentation of brain pathology in patients with milder injury. However, it is often impractical and not cost-effective in the acute setting. This patient has MTBI, and observation for a few hours and possibly a CT scan to rule out contusions are appropriate. He does not have severe enough trauma to warrant admission or invasive monitoring of his ICP. (Answer: A—Continue with observation and repeated neurologic examinations; repeat assessment with the GCS periodically; and consider imaging with a CT scan to rule out contusions)

25. A 46-year-old woman is brought to the emergency department by EMS after being involved in a car accident. She was a passenger in the back seat of the car. The accident involved frontal impact, with the car moving at 50 mph. The patient was not wearing a seatbelt. The driver says she has not been awake since the accident, which occurred 30 minutes ago. On admission, the patient’s vital signs are as follows: blood pressure, 100/60 mm Hg; heart rate, 78 beats/min; respiratory rate, 8 breaths/min; GCS score, 7. A CT scan shows a frontal epidural hematoma with mass effect.

How would you treat this patient?

A. Intubate the patient, administer hyperventilation to a carbon dioxide tension (PCO2) of 25 to 35 mm Hg, induce a barbiturate coma, and admit the patient to the ICU for further evaluation
B. Intubate the patient, administer hyperventilation to a PCO2 of 25 to 35 mm Hg, and ask for emergent neurosurgery consult for evacuation of the hematoma
C. Intubate the patient, administer hyperventilation to a PCO2 of 25 to 35 mm Hg, admit to ICU for close observation, and consult neurosurgery for intraventricular ICP monitoring
D. Admit to ICU for further evaluation and start mannitol and steroids

Key Concept/Objective: To understand the treatment of severe head injury

In patients with severe brain injury, the first priority should be cardiopulmonary resuscitation. Comatose patients with TBI are often hypoxic or hypercapnic, even though ventilation may appear to be normal. Patients who are in a coma (GCS score of less than 8) should undergo gentle hyperventilation via intubation until a PCO2 of about 35 mm Hg is achieved. Short-term hyperventilation to levels of about 25 mm Hg can be lifesaving in the patient with impending herniation. Subdural and epidural hematomas should be evacuated promptly when associated with a significant mass effect, because it has been shown that there is a significant poorer outcome with surgical delays of greater than 4 hours. The literature supports a standard recommendation that corticosteroids should not be used for neuroprotection or control of ICP in patients with severe TBI. (Answer: B—Intubate the patient, administer hyperventilation to a PCO2 of 25 to 35 mm Hg, and ask for emergent neurosurgery consult for evacuation of the hematoma)

26. A 22-year-old man is transferred to your hospital from a local hospital, where he presented 3 hours ago with closed head trauma. He lost consciousness for 10 minutes. At the first hospital where he was taken, he was given pain medications, and a CT scan was performed; the CT scan was negative. The patient is awake and complains only of moderate headache. His physical examination is unremarkable. The family is concerned about the development of seizures in the future, because they had a relative who had that problem.

What would you recommend regarding prophylaxis for seizures in this patient?
A. Phenytoin for 1 to 2 weeks
B Carbamazepine for 6 months
C. Obtain an electroencephalogram; if it is abnormal, start phenytoin
D. Do not start any antiseizure medication at this time

Key Concept/Objective: To understand the evaluation of the risk of posttraumatic epilepsy

The risk of epilepsy in patients with closed-head injury is relatively small: 2% to 5% in all patients and about 10% to 20% in patients with severe closed-head injury. A higher incidence of seizures has been seen in patients with depressed skull fractures (15%), hematomas (31%), and penetrating brain wounds (50%). This patient has a mild, closed injury, and he is at very low risk for developing seizures. Because most patients who develop posttraumatic epilepsy in the first week after injury will have recurrent seizures for some time, anticonvulsant therapy is indicated in documented cases. Controlled, randomized studies have shown that the use of phenytoin, phenobarbital, carbamazepine, and valproate do not prevent the development of posttraumatic epilepsy beyond the first week after injury. It is recommended as a standard of care that these medications should not be used to prevent posttraumatic epilepsy in patients who have not had a seizure. (Answer: D—Do not start any antiseizure medication at this time)

27. A 44-year-old woman presents to your clinic complaining of persistent problems since being in a car accident 2 years ago. At the time of the accident she suffered moderate head trauma, which required admission to a hospital for 3 days. Since then, she has felt as if she is not the same person. She has had problems with her husband, and she feels sad all the time. She also has lost interest in social activities, and she has lost 12 lb. Her thought process seems slow since the accident. Her physical examination is unremarkable.

What is the most likely diagnosis, and how would you treat this patient’s symptoms?
A. The patient probably has a personality disorder; she should not have sequelae from accidents of this nature; refer for psychotherapy
B. The patient probably has an undisclosed substance abuse problem; refer to psychiatry
C. The patient probably has neuropsychiatric sequelae from the accident; educate her about the possible sequelae, and start a selective serotonin reuptake inhibitor (SSRI) for depression
D. The patient probably has major depression, but it is unlikely that this depression is related to the accident; start an SSRI

Key Concept/Objective: To understand neuropsychiatric sequelae of head trauma

The neuropsychiatric sequelae of brain injury, both socially and in the workplace, are well appreciated. Neurologic abnormalities may not be as distressing to the patient or the patient’s family as personality changes and inappropriate behavior. Suitable treatment of neurobehavioral sequelae will often decrease patient and caregiver distress and markedly improve overall outcome. The SSRIs are favored because they are safe and easy to administer. Other common neuropsychiatric sequelae are irritability, aggression, attention deficits, seizures, memory problems, and posttraumatic stress disorder. (Answer: C—The patient probably has neuropsychiatric sequelae from the accident; educate her about the possible sequelae, and start a selective serotonin reuptake inhibitor [SSRI] for depression)

For more information, see Salazar AM: 11 Neurology: V Traumatic Brain Injury. ACP Medicine Online (www.acpmedicine.com). Dale DC, Federman DD, Eds. WebMD Inc., New York, April 2003

Neoplastic Disorders

28. A 43-year-old African-American man with known HIV disease presents to the emergency department with a 2-week history of progressive, frontal headache and associated blurry vision. Two days before admission, he developed generalized tonic-clonic seizures. The patient denies having fever, chills, or weight loss. He occasionally uses alcohol and has smoked two packs of cigarettes a day for 25 years; he admits to having used intravenous drugs in the remote past. Physical examination reveals right homonymous hemianopsia and generalized brisk deep tendon reflexes. External ocular movements are intact. Papilledema is not present, nor does the patient have any motor or sensory defects. A firm, mobile, right supraclavicular lymph node measuring 2 cm is noted. The patient’s laboratory results are as follows: Hb, 9.5 g/dl; Hct, 28%; renal function test results are normal; liver function test results are normal, with the exception of an albumin value of 3.0 g/dl; LDH, 365 mg/dl; CD4+ T cell count, 260 cells/µl; viral load, 45,000 copies/ml; serum cryptoccocal antigen is negative. Chest radiography reveals widening of the mediastinum, and right paratracheal adenopathy is present. An abrupt cutoff of the right main-stem bronchus is noted. MRI of the head shows large right parietal and right occipital lesions with ring enhancement and significant surrounding edema. No midline shift is noted. Multiple contrastenhancing ring lesions are observed in the bilateral frontal lobes. CT of the chest shows a large mediastinal mass encroaching on the right mainstem bronchus. A speculated mass measuring 2 cm is also noted at the right upper lobe.

Which of the following diagnoses is most likely?
A. Primary CNS tumor
B. Primary CNS lymphoma
C Metastatic lung cancer
D. Cryptoccocal meningitis
E. Metastatic prostate cancer

Key Concept/Objective: To understand the relationship of immunosuppression with the risk of malignancy

This patient has HIV disease with poor control of viral burden. However, his CD4 count is relatively preserved. As such, he is at negligible risk for a number of opportunistic infections, including primary CNS lymphoma and toxoplasmosis. He is still at risk for cryptoccocal disease; however, a negative serum antigen coupled with the presence of extensive intrathoracic disease makes this diagnosis very unlikely.
Primary CNS tumors are much less common than metastatic disease to the CNS. Astrocytomas and oligodendrogliomas more commonly present as infiltrating lesions. Meningiomas arise from the dura mater and are almost always solitary masses. Acoustic neuromas can be single or bilateral, but they affect the eighth cranial nerve.
The radiologic characteristics in this patient favor the diagnosis of metastases to the CNS. Tumors that frequently metastasize to the CNS include tumors of the breast and lung and melanomas. Prostate cancer almost never metastasizes to the brain. The presence of extensive intrathoracic disease, the history of tobacco exposure, and the MRI pattern support the diagnosis of lung carcinoma. (Answer: C—Metastatic lung cancer)

29. A 55-year-old white woman with known breast cancer that was treated 10 years ago with mastectomy comes for evaluation of rhythmic movement of the right arm. These episodes occurred on three occasions over the past 2 weeks; each episode lasted 5 to 10 minutes. No loss of consciousness or incontinence of the bladder or bowel was associated with the episodes. The patient denies having headache, blurry vision, or diplopia. Her family notes that the patient seems less prone to engage in conversation and seems to be sleeping more than usual. No nausea or vomiting is noted. On examination, the right breast is normal. A prosthesis of the left breast is noted; otherwise, the physical examination is unremarkable. No axillary or supraclavicular adenopathy is noted. The patient’s neurologic function is intact. MRI shows a 2 cm mass adjacent to the frontal lobe. The lesion exhibits the same density as the surrounding brain parenchyma. Minimal edema is noted. No other lesions are appreciated.

Which of the following is the most likely diagnosis?
A. Metastatic disease from breast cancer
B. Astrocytoma
C. Oligodendroglioma
D. Schwannoma
E. Meningioma

Key Concept/Objective: To understand the relationship between meningioma and breast cancer and the radiologic characteristics of different CNS tumors

Breast cancer is known to metastasize to the CNS. As with other metastatic tumors, breast cancer tends to produce multiple lesions that are most commonly located at the junction of the white matter and gray matter. These lesions are characteristically surrounded by a significant amount of edema; occasionally, the edematous area is out of proportion with the size of the metastasis. In this patient, the amount of time that has elapsed since her mastectomy makes this possibility unlikely, although tumor recurrence after 10 years has been reported. Both astrocytomas and oligodendrogliomas are tumors situated within the brain parenchyma. Both types tend to present as solitary masses without clearly defined margins. Edema, although frequently present, is less significant than the cerebral edema associated with metastatic disease.
Schwannomas occur in the cranium or peripheral nerves. Schwann cells produce myelin, which accounts for why these tumors are adjacent to nerves. Schwannomas of the eighth cranial nerve are known as acoustic neuromas. They usually present with unilateral deafness, unilateral tinnitus, or both. When they occur bilaterally, they are always associated with type 2 neurofibromatosis, a disease caused by a deletion of the tumor suppressor gene located in the short arm of chromosome 22. In this patient, the location of the tumor and the absence of deafness and tinnitus effectively exclude this diagnosis. Meningiomas arise from the dura and the arachnoid villa of intracranial and spinal spaces. These are slow-growing tumors; patients usually present with either symptoms of a space-occupying lesion or seizures of new onset. Radiologically, these tumors are characterized by their extraparenchymal location and the fact that they have a density similar to that of surrounding brain tissue. Of interest, women with breast cancer are known to have an increased incidence of meningiomas. Prognosis of meningiomas is in general excellent; surgical excision tends to be curative. Meningiomas that are difficult to excise completely (e.g., those located at the base of the brain) or that have anaplastic features are more likely to recur. For such patients, postsurgical radiation therapy is recommended. (Answer: A— Metastatic disease from breast cancer)

30. A 45-year-old white woman presents to the office for evaluation; she has been having difficulty speaking and her gait has been unsteady. Both symptoms were first noted 1 month ago and have progressed since then. The patient has not experienced fever, chills, headache, diplopia, or weight loss. The patient does not have a history of alcohol abuse, and she denies having numbness, tingling, or any weakness. Physical examination reveals ataxia of both upper extremities and both lower extremities. Ataxic gait is noted. All laboratory tests are normal. No masses are identified on chest x-ray. MRI of the head shows no evidence of cerebellar lesions, hemorrhage, or atrophy.

Which of the following antibodies are most likely to be present in the serum of this patient?

A. Anti-Hu
B. Anti-Yo
C. Anti-MAG
D. Antibody against acetylcholine receptor

Key Concept/Objective: To understand the relationship between symptoms of primary malignancy and the diagnosis of paraneoplastic syndrome and to understand the clinical characteristics of cerebellar degeneration

This case illustrates how the indirect manifestations of primary malignancies can precede the direct manifestations of those malignancies. Such disorders are known as paraneoplastic syndromes. Examples of such manifestations include deep vein thrombosis, nephritic syndrome, polycythemia, and neurologic manifestations. Certain tumor antigens are similar to native antigens. Antibodies to the malignant tumor cross-react with native antigen, giving rise to these clinical syndromes. It is well recognized that paraneoplastic syndromes can precede by weeks to months the clinical presentation of the underlying primary malignancy. As such, awareness of these syndromes is of great importance, and maintaining a high index of suspicion could lead to an earlier diagnosis.
Patients with Lambert-Eaton syndrome present with weakness. The symptoms get better during the day (unlike the symptoms of myasthenia), and repetitive use of the affected limb increases the strength of that limb. An antibody against the acetylcholine receptor is responsible for this paraneoplastic syndrome. Small cell carcinoma is most often found to be the underlying tumor. The peripheral nervous system is affected by two different sets of antibodies. Both peripheral neuropathies are predominantly sensory. In patients with lymphoma and Waldenström disease, myelin-associated glycoprotein antibodies (anti-MAG) are produced. Anti-Hu antibodies are found in patients with peripheral neuropathy and encephalomyelitis associated with small cell carcinoma of the lung. This patient has clear cerebellar signs and symptoms. A main characteristic that points toward a paraneoplastic syndrome is the bilateral nature of the findings. Furthermore, cerebellar changes in imaging were detected several months after the onset of symptoms. Patients with cerebellar tumors tend to present with unilateral signs and symptoms and abnormal neuroimaging studies. Cerebellar hemorrhage presents in a more acute manner. Alcohol abuse is associated with bilateral findings, although truncal ataxia frequently dominates the clinical picture. MRI usually demonstrates severe atrophy. The fact that this patient does not use alcohol and the normal findings on MRI argue strongly against this diagnosis. Because of the clear relationship between ovarian cancer and paraneoplastic cerebellar degeneration, this patient should undergo evaluation for this malignancy. (Answer: B—Anti-Yo)

For more information, see Posner JB: 11 Neurology: VI Neoplastic Disorders. ACP Medicine Online (www.acpmedicine.com). Dale DC, Federman DD, Eds. WebMD Inc., New York, August 2001

Anoxic, Metabolic, and Toxic Encephalopathies

31. A 45-year-old man with a history of hypertension and alcohol abuse and dependence presented to the emergency department with confusion. The patient was oriented only to person and was easily distracted. Results of physical examination were as follows: temperature, 99.2° F (37.3° C); heart rate, 88 beats/min; blood pressure, 143/88 mm Hg; and respiratory rate, 16 breaths/min. On questioning, the patient was confused and mildly agitated. The remainder of the physical examination was largely unrevealing; there were no signs of chronic liver disease and no focal neurologic findings. Laboratory evaluation was significant for a serum sodium level of 112 mEq/L and a normal serum ammonia level. The patient was admitted for further evaluation, and 3% NaCl was initiated to correct his hyponatremia. The following day, the serum sodium level was 135 mEq/L. After showing initial clinical improvement in alertness and cognition, the patient’s clinical status declined on hospital day 4. He has become obtunded and has developed flaccid quadraparesis and extensor plantar responses.

Which of the following conditions most likely accounts for the change in this patient’s status?
A. Portosystemic encephalopathy
B. Wernicke-Korsakoff syndrome
C. Central pontine myelinolysis
D. Delirium tremens

Key Concept/Objective: To understand that rapid correction of hyponatremia can lead to central pontine myelinolysis

Hyponatremia and hypernatremia have several causes. Rapid changes in serum sodium concentration can cause encephalopathy because the osmotic equilibrium between the cerebral spinal fluid and other body fluids is altered. Disturbances of cognition and arousal occur and may lead to coma. Associated features include myoclonus, asterixis, tremulousness, and seizures. Seizures often respond poorly to anticonvulsant medication unless the associated metabolic disturbance has been corrected. Focal motor deficits (e.g., hemiparesis) can occur with hyponatremia in the absence of any structural lesion or can occur with hypernatremia as a result of intracerebral or subdural hemorrhage related to osmotically caused brain shrinkage, with secondary tearing of blood vessels. Hyponatremia should be corrected at a rate not exceeding 12 mEq/L/day because rapid correction of hyponatremia leads to central pontine myelinolysis. Central pontine myelinolysis may obscure or follow improvement in hyponatremic encephalopathy. The pathologic hallmark of the disorder is breakdown and loss of myelin in the anterior pons and other brain stem regions, which may be visualized by magnetic resonance imaging. The disorder is associated with alcoholism, electrolyte disturbances, malignant disease, and malnutrition, and it relates particularly to the rapid correction of hyponatremia. (Answer: C—Central pontine myelinolysis)

32. A 78-year-old woman is transported to the emergency department after being “found down” by a family member. Upon arrival at the emergency department, the patient is pulseless and apneic. A “code 10” is called, and advanced cardiac life support is initiated. Chest compressions are performed, and the patient is intubated and oxygenated with 100% fraction of inspired oxygen (FIO2). Cardiac monitoring reveals asystole. Pharmacologic therapy with epinephrine and atropine is administered. After the second round of epinephrine and atropine, the patient regains a pulse. She is transferred to the medical intensive care unit for further care. You are concerned about the possibility of anoxic-ischemic encephalopathy secondary to circulatory arrest.

Which of the following statements regarding anoxic-ischemic encephalopathy is accurate?
A. In the mature nervous system, white matter is generally more vulnerable to ischemia than gray matter
B. In the mature nervous system, the brain stem is more vulnerable to ischemia than the cerebral cortex
C. The persistent vegetative state is characterized by the return of the sleep-wake cycle, but wakefulness is without awareness
D. Brain death is defined as the loss of all cerebral activity for at least 48 hours

Key Concept/Objective: To understand the characteristics of the persistent vegetative state and the definition of brain death

The persistent vegetative state is characterized by the return of sleep-wake cycles and of various reflex activities, but wakefulness is without awareness. Recent studies have indicated that the minimally conscious state, which is characterized by inconsistent but clearly discernible behavior of consciousness, can be distinguished from coma and a vegetative state by the presence of behavioral conditions not found in either of those two conditions; this distinction is important because outcome appears to be different in minimally conscious patients. Brain death is defined as the loss of all cerebral activity, including activity of the cerebral cortex and brain stem, for at least 6 hours if confirmed by electroencephalographic evidence of electrocerebral inactivity or for 24 hours without a confirmatory electroencephalogram. (Answer: C—The persistent vegetative state is characterized by the return of the sleep-wake cycle, but wakefulness is without awareness)

33. A 68-year-old woman with a history of alcohol abuse and dependence presents to the primary care clinic for evaluation of confusion. The patient is accompanied by her daughter, who is concerned about her mother’s forgetfulness and who feels that her mother has been “making things up.” Physical examination is significant for nystagmus, ophthalmoplegia, and ataxic gait. On the basis of the physical examination and a history of confabulation, you make the diagnosis of Wernicke encephalopathy.

A deficiency of which of the following is responsible for this condition?

A. Vitamin B12 (cyanocobalamin)
B. Folic acid
C. Niacin
D. Thiamine (vitamin B1)

Key Concept/Objective: To understand that thiamine (vitamin B1) deficiency is responsible for Wernicke encephalopathy

Thiamine (vitamin B1) deficiency is responsible for the hallmark features of Wernicke encephalopathy. These features include ophthalmoplegia, gait ataxia, and fluctuating confusional states. Pathologic changes occur in characteristic regions of the brain stem, especially in the mamillary bodies and thalamus. Diffusion-weighted magnetic resonance imaging may show signal changes in these characteristic midline locations. As with Wernicke encephalopathy, Korsakoff encephalopathy is attributed to thiamine deficiency, though the precise pathophysiology is unknown. Selective disturbance of memory is the predominant clinical abnormality in Korsakoff encephalopathy. Thiamine replacement therapy rarely leads to improvement. There is marked impairment of recent memory and difficulty in incorporating new memories, though immediate recall is intact. Patients are unaware of any deficit and often confabulate. Other cognitive abnormalities are found less often. The disorder is common in chronic alcoholics, often occurring in association with Wernicke encephalopathy. The pathologic changes are similar in distribution to those in Wernicke encephalopathy. (Answer: D—Thiamine [vitamin B1])

For more information, see Aminoff MJ: 11 Neurology: VII Anoxic, Metabolic, and Toxic Encephalopathies. ACP Medicine Online (www.acpmedicine.com). Dale DC, Federman DD, Eds. WebMD Inc., New York, January 2003

Headache

34. A 25-year-old woman comes to your clinic complaining of headaches. She’s been having unilateral headaches for about a year, at a rate of approximately two to three episodes a month. The headaches last for 6 to 8 hours, are pulsating, and are accompanied by nausea, vomiting, and photophobia. She also has been experiencing some rhinorrhea with the headaches. The pain is moderate in intensity and gets worse with routine physical activity. She usually gets relief by taking over-the-counter ibuprofen. Her medical history is positive for sinus allergies. She has a family history of headaches. Her physical examination, including a neurologic examination, is unremarkable. She has heard on the news that headaches can be a sign of a tumor, and she is concerned about this possibility.

On the basis of this patient’s history and physical examination, what is the most likely diagnosis, and how would you evaluate her?
A. Tension-type headache; further workup is not indicated at this time
B. Sinus headache; obtain a sinus computed tomographic scan
C. Migraine without aura; no further workup is indicated at this time
D. Migraine without aura; magnetic resonance imaging is indicated to rule out intracranial pathology

Key Concept/Objective: To understand the clinical characteristics and evaluation of migraines

Migraine can occur with and without aura. According to International Headache Society criteria for migraine without aura, the duration of untreated or unsuccessfully treated episodes ranges from 4 to 72 hours. The headaches are associated with at least two of the following pain characteristics: unilateral location, pulsating quality, moderate or severe intensity, and aggravation by routine physical activity. The pain is accompanied by at least one of the following symptoms: nausea with or without vomiting, photophobia, and phonophobia. Forty-five percent of migraineurs have at least one autonomic symptom (i.e., lacrimation, eye redness, or rhinorrhea) during an attack; these symptoms can lead to confusion of migraine with sinus headaches. Tension-type headaches are usually nonpulsating, last from 30 minutes to 7 days, are mild or moderate in severity, and are bilateral in location. There should be no nausea or vomiting; either photophobia or phonophobia may be present, but not both. The vast majority of patients require no diagnostic testing at all; they can be diagnosed accurately on the basis of a detailed history and a physical examination. (Answer: C—Migraine without aura; no further workup is indicated at this time)

35. A 38-year-old man with a 15-year history of migraine presents to your clinic complaining of worsening headaches. He has been taking ibuprofen. The headaches are unilateral and are accompanied by nausea, vomiting, and photophobia. In the past, the patient obtained relief with ibuprofen, but over the past few months, the headaches have failed to respond to this medication. The only change in the characteristics of the headaches is that they have increased in frequency. He used to have two or three attacks a week, and now he has headaches 5 or 6 days a week. His physical examination is unremarkable.

What is the most likely explanation of this patient’s worsening headaches, and how would you treat these headaches?
A. Misdiagnosed tension-type headaches; start a tricyclic antidepressant as a prophylactic agent
B. Worsening migraines; start triptans
C. Brain tumor; obtain imaging studies and refer to neurosurgery
D. Chronic daily headache (CDH) from a transformed migraine; stop short-acting nonsteroidal anti-inflammatory drugs (NSAIDs) (e.g., ibuprofen)

Key Concept/Objective: To understand CDH

CDH has a frequency of 15 or more days a month. CDH includes four different headache types: transformed migraine, chronic tension-type headache, hemicrania continua, and new drug persistent headache. Transformed migraine, with or without medication overuse, is a complication of intermittent migraine that usually occurs by 20 to 30 years of age. In 80% of patients, there is a gradual transformation from episodic migraine to CDH that may be associated with analgesic overuse and psychological factors. Migraine characteristics are present to a significant degree. Migraineurs are particularly susceptible to rebound headaches, which can occur with frequent use of symptomatic medications, including acetaminophen, aspirin, caffeine, NSAIDs with short half-lives, butalbital, ergotamine, opiates, and triptans. In the treatment of these patients, these medications should be discontinued. Usually the headaches improve within 9 days. A migraine-preventive medication can also be started. Two strategies that can be used during this withdrawal period are the use of steroids and, alternatively, a combination of tizanidine and an NSAID with a long half-life. Chronic tension-type headache presents with a frequency of 15 or more a month for at least 6 months. The pain characteristics are the same as for episodic tension-type headaches. The possibility of a brain tumor is less likely in this patient, given the similarity of his current headaches to his previous headaches, the temporal progression, and the presence of a more likely explanation. (Answer: D—Chronic daily headache [CDH] from a transformed migraine; stop short-acting nonsteroidal anti-inflammatory drugs [NSAIDs] [e.g., ibuprofen])

36. A 48-year-old man presents to a walk-in clinic complaining of headaches. The headaches are unilateral, located behind his right eye; they are constant, severe, and are of rapid onset. The patient notes that the headaches are accompanied by right-sided nasal congestion. The headaches usually last for 30 to 40 minutes. The headaches started 3 or 4 weeks ago, and he has two or three attacks a day. The patient does not have a history of headaches. At this time, the headache is 8/10 in severity. The patient has a medical history of hypertension, for which he takes an angiotensin-converting enzyme (ACE) inhibitor. On physical examination, the patient’s blood pressure is 150/78 mm Hg; right-sided ptosis, miosis, and conjunctival injection are present. The rest of the examination is normal.

What is the most likely diagnosis for this patient, and how would you approach the diagnosis?

A. Cluster headaches; consider obtaining an imaging study to evaluate for secondary headaches
B. Hypertension; increase dosage of blood pressure medications
C. Posterior communicating artery aneurysm; get an urgent cerebral angiography
D. ACE inhibitor–induced headache; stop use of the ACE inhibitor

Key Concept/Objective: To know the clinical manifestations of cluster headaches

Cluster headaches are an uncommon headache type that occurs in only about 0.4% of the general population. Cluster headaches are five times more common in males. This condition is marked by periods of recurrent headaches (one to eight a day) interspersed with periods of remission. Cluster headaches are unilateral and severe. The most common types of pain are orbital, retroorbital, temporal, supraorbital, and infraorbital. The headache may alternate sides. Cluster headaches have a rapid onset, with peak intensity in 5 to 10 minutes, and are usually of short duration, lasting 30 to 45 minutes. Autonomic symptoms are present in over 97% of the cases. Lacrimation and conjunctival injection are each present in 80% of the cases, and ipsilateral congestion is present in 74%. A partial Horner syndrome is present in 65% of the cases. Cluster headaches can usually be diagnosed on the basis of the clinical criteria alone. Neuroimaging, preferably MRI, may be considered in cases with the following features: a pattern of clusterlike headache not conforming to the clinical criteria; age of onset older than 40 years; a progressive pattern of headaches; chronic cluster headache; and any focal neurologic deficit other than Horner syndrome. A posterior communicating artery aneurysm can cause a subarachnoid hemorrhage and pupillary changes characteristic of third-nerve palsy (i.e., midriasis with ptosis); it would not cause miosis. Mild to moderate hypertension does not usually cause headache. ACE inhibitors can cause headaches in some patients; however, in this patient, this possibility seems less likely than cluster headaches. (Answer: A—Cluster headaches; consider obtaining an imaging study to evaluate for secondary headaches)

37. You are asked to evaluate a 25-year-old woman for headaches. Her symptoms started 2 months ago with daily frontal bilateral headaches. The headaches are pulsatile and continuous. She also complains of occasional blurred vision. She has no significant medical history. She takes over-the-counter acetaminophen for her headaches. On physical examination, her blood pressure is 120/76 mm Hg. She weighs 200 lb, and she is 5 ft 2 in tall. Her fundoscopic examination shows papilledema. The rest of her examination is normal. You are concerned about her symptoms and order an MRI, which shows no significant abnormalities.

What should be your next step in the management of this patient?
A. Perform a lumbar puncture
B. Start a triptan
C. Start indomethacin
D. Start a beta blocker as prophylaxis for migraines

Key Concept/Objective: To recognize the manifestations of pseudotumor cerebri

Pseudotumor cerebri, also known as idiopathic intracranial hypertension, is a disorder of unknown etiology. Onset usually occurs in persons between the ages of 11 and 58 years. Ninety percent are young, obese women. Headache is present in 75% or more of patients, papilledema in 95%, cranial nerve VI palsy in 25%, transient visual obscurations in 70%, visual loss in 30%, and roaring noises in 70%. The headaches are usually pulsatile, continuous, and daily; they can be unilateral or bilateral, with a bifrontotemporal location being the most common. Nausea is present in 60% of cases, and vomiting is present in 40%. The diagnosis of pseudotumor cerebri is one of exclusion, because there are many other causes of papilledema. Testing includes a scan of the brain; MRI is the test of choice. If the brain scan is negative, a lumbar puncture should be obtained. The opening pressure is usually elevated and the cerebrospinal fluid analysis is normal, except for a low CSF protein level in some patients. Treatments include weight loss and diuretics to decrease the CSF production. (Answer: A—Perform a lumbar puncture)

For more information, see Evans RW: 11 Neurology: VIII Headache. ACP Medicine Online (www.acpmedicine.com). Dale DC, Federman DD, Eds. WebMD Inc., New York, April 2003

Demyelinating Diseases

38. A 54-year-old man comes to the hospital complaining of left arm weakness of 3 days’ duration. He has no significant medical history. He takes no medications and denies using alcohol or tobacco. His physical examination is unremarkable except that his motor examination shows 3/5 strength in the left upper extremity. Results of the neurologic examination are within normal limits. MRIs of the brain are obtained. T2-weighted MRI shows a single hyperintense white matter lesion on the right hemisphere; the lesion is hypointense on T1-weighted MRI.

On the basis of this patient’s symptoms and imaging studies, which of the following would be the most appropriate step to take next in his management?
A. Start glatiramer for treatment of multiple sclerosis (MS)
B. Start interferon beta for treatment of MS
C. Perform a lumbar puncture, obtain a spinal MRI, and assess evoked responses
D. Start mitoxantrone to prevent progression of symptoms

Key Concept/Objective: To understand the diagnosis of MS

The diagnosis of MS is based on clinical signs and symptoms, MRI findings, and other laboratory tests. The diagnosis of MS requires evidence of dissemination of lesions in time and space and the careful exclusion of other causes. The patient should have had more than one episode of neurologic dysfunction and should have evidence of white matter lesions in more than one part of the CNS. MRI is the single most useful laboratory test in the diagnosis of MS. Although MRI is extremely sensitive in detecting white matter lesions in patients with MS, it is not very specific. Many other diseases produce white matter lesions. Therefore, MRI findings should never be used as the sole basis for the diagnosis. The most characteristic abnormality of the CFS in MS is the presence of intrathecal synthesis of immunoglobulins of restricted heterogeneity, which is demonstrated by the presence of oligoclonal immunoglobulin bands. Evoked responses are a useful marker of subclinical MS and can be used in support of the diagnosis of MS. Interferon beta, glatiramer, and mitoxantrone have been used in MS; however, the specific indications for treatment with these agents are not met in this case. (Answer: C—Perform a lumbar puncture, obtain a spinal MRI, and assess evoked responses)

39. A 40-year-old woman with a history of MS presents to clinic with numbness in her left arm and weakness in her right leg. She was diagnosed 2 years ago after having an episode of left leg weakness. She has experienced mild, persistent weakness since then. She had optic neuritis 3 years ago. Her physical examination is remarkable for severe left arm weakness and increased deep tendon reflexes of the left arm. The left leg shows mild weakness and increased reflexes. She has decreased sensation in the right leg. MRI shows multiple white matter lesions.

On the basis of this patient’s history and symptoms, which of the following would be the most appropriate therapeutic regimen?
A. Short-term steroids
B. Long-term mitoxantrone
C. Long-term steroids
D. Long-term glatiramer or interferon beta and short-term steroids

Key Concept/Objective: To understand therapy for MS

In MS, the management of acute relapses varies with the severity of the presenting symptoms and signs. High-dose corticosteroid therapy is indicated for exacerbations that adversely affect the patient’s function. A short, tapering course of corticosteroids may be given afterward. In the past few years, three different medications that affect the long-term clinical course of MS have been approved: interferon beta-1b, interferon beta-1a, and glatiramer acetate (previously known as copolymer-1). These drugs reduce the frequency of attacks and limit the accumulation of fixed lesions on MRI. In patients with relapsingremitting MS, these agents may delay the accumulation of disability. The choice of which agent to use depends on the particular patient. Patients who are maintained on these therapies can expect an 18% to 50% reduction in attack frequency. Best responses with any of the available drugs appear to result from initiation of treatment relatively early in the disease course. Mitoxantrone currently has a role for selected patients with very active disease; it is approved for the treatment of aggressive relapsing and secondary progressive MS. Its significant side effects limit its use. (Answer: D—Long-term glatiramer or interferon beta and shortterm steroids)

40. A 26-year-old woman is evaluated for decreased vision and eye pain. Her symptoms started 2 days ago with pain in her right eye with ocular movements. Over the past 24 hours, she has experienced a decrease in central vision in her right eye. She has no significant medical history. Physical examination shows decreased central vision and pain on ocular movement of the right eye. There is an afferent pupillary defect on the right. Results of fundoscopic examination are normal. The rest of her examination is unremarkable. MRIs of the brain and spinal cord are consistent with optic neuritis on the right; there are two white matter lesions in the periventricular area.

On the basis of this patient’s presentation and MRI findings, which of the following statements is most accurate?
A. The patient has MS and should be started on steroids and glatiramer
B. The patient has optic neuritis; she is at significant risk for developing MS in the future
C. The patient has optic neuritis; she is at no risk of progressing to MS in the future
D. The patient has MS; she should be started on mitoxantrone

Key Concept/Objective: To know the association between optic neuritis and MS

Optic neuritis is an acute inflammatory optic neuropathy. The cardinal symptoms are unilateral vision loss and retrobulbar pain with eye movement. Treatment with intravenous methylprednisolone followed by oral prednisone hastens recovery of vision. Even without treatment, almost all patients begin to recover vision within 4 weeks. The relationship of optic neuritis to MS is controversial. Some regard optic neuritis as a distinct entity, but others consider it part of the clinical continuum of MS. More than half of all patients with MS have optic neuritis at some time during the course of disease. Of patients who present with optic neuritis and who have no other neurologic deficit, almost 40% have one or more ovoid periventricular lesions on brain MRI; clinically definite MS eventually develops in
60%. Patients with completely normal results on MRI and comprehensive CSF evaluation seldom progress to MS. (Answer: B—The patient has optic neuritis; she is at significant risk for developing MS in the future)

41. A 44-year-old man comes to the hospital complaining of progressive lower extremity weakness and decreased sensation. He has been experiencing these symptoms for 5 days. He also complains of having difficulties with bowel movements and urination. He recalls having an upper respiratory infection 1 or
2 weeks ago. His physical examination is remarkable for decreased sensation starting at the level of T10, symmetrical severe lower extremity weakness, urinary retention, and decreased rectal tone. The muscle tone and deep tendon reflexes in his lower extremities are diminished. T2-weighted MRI of the spinal cord shows a hyperintense lesion that involves the majority of the cross-sectional area of the cord; the lesion extends from T6 to L3. No cord compression is seen. MRI of the brain is normal.

Of the following, which is the most likely diagnosis?
A. Devic disease
B. Acute disseminated encephalomyelitis
C. Transverse myelitis
D. MS

Key Concept/Objective: To be able to recognize transverse myelitis

Acute transverse myelitis is a syndrome of spinal cord dysfunction. It has a rapid onset; it may occur after infection or vaccination or it may occur with no discernible precipitant. It may also be the initial presentation of MS. Symptoms include paraparesis, which is initially flaccid and then spastic; loss of sensation with a sensory level in the trunk; and bowel and bladder dysfunction. The thoracic cord is most often affected. MRI is extremely helpful for excluding other structural lesions and for confirming the presence of an intramedullary lesion, which is typically hyperintense in T2-weighted imaging. No treatment has proven to be beneficial, but corticosteroids are often used. Neuromyelitis optica is also known as Devic disease. It is characterized by the simultaneous or sequential involvement of the optic nerves and spinal cord; it often has a malignant course. Acute disseminated encephalomyelitis is a monophasic syndrome that is usually preceded by a viral exanthema, an upper respiratory infection, or vaccination. Onset is rapid and is characterized by meningeal signs, headache, seizures, and altered mental status. The neurologic deficits include hemiplegia, paraplegia, sensory loss, vision loss, and transverse myelitis. In this patient, the lack of multiple lesions and the monophasic nature of the disease make the diagnosis of MS less likely. (Answer: C—Transverse myelitis)

42. A 43-year-old woman with a 14-year history of MS with lower-extremity spasticity presents with increased spasticity, mild confusion, abdominal discomfort, and a temperature of 100° F (37.8° C).

Which of the following choices best combines medical indication and favorable cost-benefit ratio?
A. Urodynamic study
B. Abdominal CT scan
C. MRI scan of the lumbosacral spine
D. Brain MRI
E. Urinalysis

Key Concept/Objective: To understand the frequency of UTIs in women with MS and how acute UTI can mimic relapse of acute MS

Bladder dysfunction is common in patients with MS; in women especially, bladder dysfunction often results in UTIs. Because of loss of sensation, the infections may not cause dysuria but may instead cause more global deterioration of neurologic function, mimicking an acute relapse. Lower limb spasticity in particular may accompany urinary retention with overflow incontinence. In this patient, as in others who appear to have acute relapse, a urinalysis should be part of the evaluation. A formal urodynamic study will define the specific pattern of dysfunction and help determine appropriate therapy. (Answer: E— Urinalysis)

43. A 30-year-old woman presents with vision loss in the left eye and pain behind the left eye with eye movement. Ophthalmologic consultation reveals optic neuritis. Brain MRI and comprehensive CSF examination are normal.

Which of the following statements is true for this patient?
A. Treatment with high-dose methylprednisolone will result in improved visual retention at 1 year
B. This patient has a high likelihood of progression to definite MS within 2 years
C. Without treatment, this patient has a high likelihood of progressive unilateral visual loss
D. It is unlikely that this patient will progress to MS
E. Optic neuritis is distinct from MS

Key Concept/Objective: To understand the relation between optic neuritis and multiple sclerosis

Although more than half of all patients with MS have optic neuritis at some time, patients with optic neuritis who have completely normal results on MRI scanning and comprehensive CSF examination seldom progress to MS. Whether optic neuritis is a distinct clinical entity or part of a continuum with MS is controversial. Treatment of optic neuritis may result in hastened benefit, but even without treatment, most patients begin to recover vision within 4 weeks. (Answer: D—It is unlikely that this patient will progress to MS)

44. A 45-year-old man presents with paresthesias in his feet and mild ataxia. He has an uneventful medical history and has never used alcohol. On physical examination, he has decreased proprioception and vibratory sensation in the lower extremities. Routine laboratory studies show a macrocytic anemia.

Which test should be ordered next for this patient?
A. Serum cobalamin
B. CSF immunoglobulin electrophoresis
C. Visual evoked potentials
D. Serum folate
E. Brain MRI

Key Concept/Objective: To understand the neurologic consequences of vitamin B12 deficiency

In addition to causing a macrocytic anemia, vitamin B12 deficiency results in axonal demyelination, especially in the dorsal and lateral columns of the spinal cord. This leads to the common presenting symptoms of peripheral paresthesias, ascending sensory loss, and sensory ataxia. More severe and prolonged deficiency can result in memory loss and confusion. Folate deficiency can result in macrocytic anemia but does not cause the neurologic complications of B12 deficiency. (Answer: A—Serum cobalamin)

For more information, see Linsey JW, Wolinsky JS: 11 Neurology: IX Demyelinating Diseases. ACP Medicine Online (www.acpmedicine.com). Dale DC, Federman DD, Eds. WebMD Inc., New York, July 2003

Inherited Ataxias

45. The parents of a 16-year-old boy bring him to you with concerns about alcohol use. He has been experiencing slurring of speech and was twice sent home from school for falling in gym class. He reports generalized fatigue and dizziness with exercise. He denies using alcohol or drugs. The patient’s maternal uncle developed difficulty walking at a young age but died in an accident before a diagnosis was made. On examination, the patient has impaired proprioception, a staggering gait, and extensor plantar response. He has a laterally displaced sustained point of maximal impulse.

What is the most likely diagnosis for this patient?
A. Ataxia-telangiectasia
B. Friedreich ataxia
C. Neuroblastoma
D. Chiari-Arnold deformity
E. Episodic ataxia type 2

Key Concept/Objective: To know the presenting features of Friedreich ataxia

Friedreich ataxia is the most common recessively inherited ataxia. Patients often present before the age of 25 years with ataxia. Other symptoms can include dysarthria, vision problems, weakness, and dysphagia. Physical examination reveals loss of deep tendon reflexes, poor proprioception, weakness, and extensor plantar response. Phenotypic variation is not uncommon, and some patients have preserved or brisk deep tendon reflexes. Between 30% and 50% of patients develop symptomatic heart disease, including hypertrophic cardiomyopathy. (Answer: B—Friedreich ataxia)

For more information, see Subramony SH: 11 Neurology: X Inherited Ataxias. ACP Medicine Online (www.acpmedicine.com). Dale DC, Federman DD, Eds. WebMD Inc., New York, February 2004

Alzheimer Disease and Other Dementing Illnesses

46. A 79-year-old man comes to establish primary care. He is accompanied by his daughter. He has been in very good health for most of his life. His only medication is a beta blocker, which he has been been taking for moderate hypertension. He has undergone all screening examinations appropriate for his age. Over the past several months, he underwent evaluation for possible onset of dementia. The patient and his daughter agree that the patient’s memory has been worsening over the past 1 to 2 years. He easily recalls events of his childhood, but he is not able to tell you what he ate for his morning meal. He had been an avid outdoorsman, but he had to give up outdoor activities because he recently got lost in the woods for several hours. The family seeks your opinion and further workup.

Which of the following statements regarding Alzheimer disease (AD) is true?
A. AD is histopathologically defined by neurofibrillary tangles and neuritic plaques in the cerebral cortex
B. The definition of AD requires impairment in only one area of cognitive function
C. The pivotal cognitive finding in AD is retrograde amnesia
D. The prevalence of AD is approximately 1% of the United States population older than 65 years

Key Concept/Objective: To understand the general clinical features of AD

AD is histopathologically defined by neurofibrillary tangles and neuritic plaques in the cerebral cortex. The definition of dementia requires impairment in more than one area of cognitive function. In the dementia syndrome of AD, in addition to anterograde amnesia, there must be impairment in at least one of the following: language, abstract reasoning, executive function, or visuospatial processing. The pivotal cognitive finding in AD is anterograde amnesia, which represents an inability to learn new things. Persons with anterograde amnesia typically cannot keep track of the date, remember recent conversations, or remember where they set something down, and they often repeat themselves in conversation. The prevalence of AD is approximately 8% of the United States population older than 65 years. (Answer: A—AD is histopathologically defined by neurofibrillary tangles and neuritic plaques in the cerebral cortex)

47. An 80-year-old male patient of yours with AD is brought to your office by his daughter. According to the daughter, the patient’s functional status has been declining rapidly. She states that along with worsening memory, the patient has become tearful, and she feels he is hallucinating. He had been stable for the past year while receiving donepezil therapy. The patient’s functional decline and tearfulness, as well as occasional severe confusion with combativeness, are causing severe stress for both the patient and the household. The patient’s daughter asks if there are further medication options for the patient.

Which of the following statements regarding primary and secondary therapies for AD is true?
A. The cholinesterase inhibitors, such as donepezil, have shown promise as a cure for AD
B. The cholinesterase inhibitors are the only class of agents available for the primary treatment of AD
C. Treatment of depression associated with AD should be pursued as aggressively as in patients without dementia
D. For treatment of anxiety and agitation in AD, short-acting benzodiazepines and typical antipsychotics are generally recommended

Key Concept/Objective: To know the appropriate primary and secondary therapies for AD

The cholinesterase inhibitors donepezil, galantamine, and rivastigmine have been approved by the Food and Drug Administration for the treatment of AD. Clinical trials with each of these agents have shown that long-term use results in modest stabilization of cognitive and functional status for approximately 6 to 12 months, compared with no treatment. Vitamin E is often recommended for patients with AD on the basis of a study of 2 years’ duration. The glutamate modulator memantine has also been approved by the FDA for the treatment of AD. This agent is a noncompetitive receptor antagonist of N-methylD-aspartate. Several clinical trials have reported positive results with memantine in the treatment of moderate to severe dementia. Treatment of depression or anxiety in patients with AD should be pursued as aggressively as in patients without AD, with adherence to the best practices of geriatric pharmacology. Depression frequently coexists with AD and contributes to morbidity and loss of function. Treatment of anxiety presents somewhat more of a challenge in AD patients, because the agents commonly used in younger patients, the benzodiazepines, have distinctly unwanted side effects in AD patients. Drugs such as lorazepam and alprazolam can increase confusion in AD patients. The longer-acting agent clonazepam may be a better choice. Buspirone is another alternative for the treatment of anxiety in AD patients. Treatment of agitation generally requires antipsychotics. Quetiapine has the significant advantage of being much less likely to induce extrapyramidal signs than both newer and older agents. (Answer: C—Treatment of depression associated with AD should be pursued as aggressively as in patients without dementia)

48. A 69-year-old female patient whom you have been treating for many years for hypertension and dyslipidemia comes for a routine appointment. Both her hypertension and dyslipidemia have been difficult to control. She has been hospitalized on several occasions over the past few years for likely transient ischemic attacks (TIA). Her most recent TIA occurred 18 months ago. She has been on aspirin therapy for many years. Today she complains that her “mind wanders.” On questioning, she reports that her short-term memory and her ability to learn new information have declined over the past several months. She believes she has “hardening of the arteries.”

Which of the following statements regarding vascular dementia (VaD) is true?
A. The National Institute of Neurological Disorders and Stroke– Association Internationale pour la Recherche et l’Enseignement en Neurosciences (NINDS-AIREN) criteria for the diagnosis of VaD are highly sensitive
B. Though lacking sensitivity, the NINDS-AIREN criteria for the diagnosis of VaD are highly specific
C. As described in NINDS-AIREN criteria, the onset of all cases of VaD
occurs within a 3-month period following a stroke
D. VaD and AD are roughly equal in prevalence

Key Concept/Objective: To know the diagnostic criteria and prevalence of VaD

The essence of the NINDS-AIREN criteria is that (1) the onset or worsening of dementia occurred within 3 months after a clinical stroke; (2) imaging studies show evidence of bilateral infarcts in cortical regions, basal ganglia, thalamus, or white matter; and (3) neurologic examination shows focal neurologic deficits. All three criteria are required for the diagnosis. Clinical-pathologic correlation studies have shown that this definition is quite specific, meaning that patients who meet these criteria are highly likely to have VaD pathologically. However, the NINDS-AIREN criteria are very insensitive, failing to diagnose the majority of patients who prove to have VaD at autopsy. VaD may also begin insidiously, because there is a substantial percentage of VaD cases that appear to result from the accumulation of a series of so-called silent or covert infarcts. Patients in this group do not meet the diagnostic criterion of dementia temporally linked to stroke, but they do have brain infarcts, best visible with MRI. VaD is approximately one tenth to one fifth as common as AD in prevalence and incidence. (Answer: B—Though lacking sensitivity, the NINDSAIREN criteria for the diagnosis of VaD are highly specific)

49. An 84-year-old man presents to your office with his live-in son. His son states that his father has become progressively forgetful over the past several months. He now has difficulty remembering to complete even simple daily tasks. He also states that his father has become physically slowed over the same period. His father now walks slowly and with a wider-than-usual gait. In addition, he falls several times a month. The son also notes that for several years, his father has talked in his sleep and that he has “nightmares” almost every night. He is concerned that his father has “old-timer’s” disease, and he seeks your opinion.

Which of the following statements regarding dementia with Lewy bodies (DLB) is true?
A. The clinical presentation of DLB is identical to that of AD
B. The parkinsonism of DLB is consistently severe
C. As with AD, the patient with DLB has no changes in level of arousal
D. The rapid eye movement sleep behavior disorder (RBD) is highly specific for DLB

Key Concept/Objective: To know the clinical distinction of DLB and AD

The term DLB is in vogue as a label for patients who have spontaneous (i.e., not druginduced) parkinsonism, dementia, and often symptoms of disordered arousal. The cognitive disorder of DLB can closely resemble AD, but in many patients, there are some notable differences. These differences in DLB include a slightly less prominent deficit in learning and memory and more prominent difficulties with visuospatial functions, performance on timed tasks, and executive functions. The parkinsonism in DLB can range from a relatively isolated gait instability with frequent falling to a typical pattern of Parkinson disease with rest tremor, rigidity, bradykinesia, and postural instability. Patients with DLB often experience marked fluctuations in their alertness and level of arousal from one day to the next. They often sleep excessively. In RBD, patients engage in dream enactment, thrashing about in bed or talking in their sleep. RBD can often precede the dementia and the movement disorder by years and is highly specific for DLB. (Answer: D—The rapid eye movement sleep behavior disorder [RBD] is highly specific for DLB)

50. A 72-year-old woman is referred to you for evaluation of 2 to 3 years of gradual memory loss. Although she clearly has significant difficulties with memory and slowed speech, she seems surprisingly apathetic toward her condition. She has hypertension and mild osteopenia but is otherwise healthy; she has no history of head trauma. Her only medication is 12.5 mg of hydrochlorothiazide once daily. Her MiniMental State Exam (MMSE) score is 21/30; her neurologic exam is normal. Neuropsychological testing confirms cognitive dysfunction. Laboratory evaluation for other causes of dementia is negative.

Which of the following is the most appropriate diagnosis for this patient?
A. Definite Alzheimer disease
B. Probable Alzheimer disease
C. Progressive supranuclear palsy
D. Normal-pressure (communicating) hydrocephalus
E. Polypharmacy

Key Concept/Objective: To know the diagnostic criteria for Alzheimer disease

This patient has at least two areas of cognitive dysfunction (memory and speech), confirmed by neuropsychological testing. She has no evidence of other disorders, such as the motor dysfunction expected with progressive supranuclear palsy or the ataxia and incontinence associated with normal-pressure hydrocephalus. It is unlikely that a low dose of hydrochlorothiazide would be responsible for significant cognitive dysfunction, particularly in the absence of laboratory abnormalities. The patient meets criteria for probable Alzheimer disease. The diagnosis of definite Alzheimer disease requires evidence of pathologic changes in brain tissue from either biopsy or autopsy specimens. (Answer: B—Probable Alzheimer disease)

51. A 65-year-old man is brought in by his family for evaluation of altered cognitive function. The family members say that he is “normal” midday but sleeps poorly and frequently wanders about the house at night. When the patient is questioned about this, he seems confused and says that he has no recollection of these events. He is being treated for depression and has no other medical problems. Current medications include amitriptyline, 100 mg at bedtime, with 1 mg of lorazepam as needed for sleep. His MMSE score is 26/30; his neurologic examination is normal. Laboratory evaluation for other causes of dementia is negative.

Which of the following is the most appropriate diagnosis for this patient?
A. Definite Alzheimer disease
B. Probable Alzheimer disease
C. Huntington disease
D. Normal-pressure (communicating) hydrocephalus
E. Polypharmacy

Key Concept/Objective: To recognize medications as a common cause of “dementia”

This patient presents with a history of sleep disturbance with nocturnal disorientation. Although this is common in dementia syndromes, this patient is on two medications that often cause cognitive dysfunction: amitriptyline (a tricyclic antidepressant with anticholinergic activity) and lorazepam (a benzodiazepine). These medications should be changed or discontinued before considering the diagnosis of a primary dementia syndrome. (Answer: E—Polypharmacy)

52. A 78-year-old man comes in with his family to discuss the fact that he does not seem to be thinking as clearly as he once did. His family says that he frequently takes a long time to respond to simple questions and does not participate in conversations as he used to. He has a history of prostate cancer, status
5 years’ postradiotherapy. He takes 325 mg of aspirin once a day. Speaking to him, you notice that he does take a long time to answer, although his answers are usually appropriate. His speech is slowed and noticeably slurred. His MMSE score is 23/30. The neurologic examination is remarkable for expressionless facies and halting, unsteady gait. Laboratory evaluation for other causes of dementia is negative.

Which of the following is the most appropriate diagnosis for this patient?
A. Definite Alzheimer disease
B. Probable Alzheimer disease
C. Progressive supranuclear palsy (PSP)
D. Pick disease
E. Depression

Key Concept/Objective: To be able to recognize evidence of subcortical dementia

Motor dysfunction is typical of the subcortical dementias, of which PSP is the only one listed among the available answers. PSP often presents with parkinsonian symptoms, typically with prominent dysarthria and dysphagia. (Answer: C—Progressive supranuclear palsy [PSP])

53. A 28-year-old man wishes to have genetic testing for Alzheimer disease because his mother and maternal grandfather both died of the disease in their early 60s. He has one child, age 5.

Before genetic testing, what should this patient be told is the risk of transmission of the gene to his child if his test is positive?
A. 13%
B. 25%
C. 33%
D. 50%
E. 100%

Key Concept/Objective: To understand autosomal dominant mutations causing Alzheimer disease and the implications for genetic counseling

The patient’s history is strongly suggestive of autosomal dominant inheritance of earlyonset Alzheimer disease. If he is tested for APP or PS1, he should be advised that a positive test in his case indicates a 50% risk of transmission of the gene to his children. (Answer: D—50%)

For more information, see Knopman DS: 11 Neurology: XI Alzheimer Disease and Other Major Dementing Illnesses. ACP Medicine Online (www.acpmedicine.com). Dale DC, Federman DD, Eds. WebMD Inc., New York, May 2004

Epilepsy

54. A 22-year-old woman comes to your clinic after being seen in a local emergency department 1 week ago, when she presented with seizures. She denies having any previous episodes. She does not remember the episode, which was witnessed by her mother. The mother relates that she noticed that the patient had a blank stare and then, after a few seconds, she started moving her hands repetitively, “like she was washing them.” During this episode, the patient did not respond to any commands; it lasted approximately 1 minute. After this episode, the patient remained confused for about 10 minutes. The patient’s physical examination is unremarkable.

On the basis of clinical presentation, how would you classify this patient’s seizure

A. Myoclonic seizure
B. Absence seizure
C. Complex partial seizure
D. Tonic-clonic seizure

Key Concept/Objective: To understand the major classification of seizures

The International League Against Epilepsy has classified epileptic seizures on the basis of the clinical presentation and electroencephalographic criteria. The classification divides seizures into three major categories: partial, generalized, and unclassified. Partial seizures are described as either simple or complex, depending on whether consciousness remains intact or is impaired during the seizure. Simple partial seizures can be motor, sensory, autonomic, or psychic. Complex partial seizures usually begin with arrest of motion and a blank stare. Automatisms, oroalimentary behavior, or verbal utterances may occur initially or during the seizure. Most spells last only a few minutes. At the termination of the seizure, the patient may be momentarily confused, fatigued, or disoriented. Generalized seizures cause a spectrum of behavior from the nonconvulsive pattern of simple absence seizure through myoclonus to the fully developed generalized tonic-clonic seizure. Absence seizures are brief, usually lasting 10 seconds or less; the seizures are not preceded by an aura or followed by postictal effects, which helps differentiate them from complex partial seizures. Myoclonus consists of brief jerks or contractions of a specific muscle or group of muscles. Atonic seizures involve a sudden loss of postural tone. Convulsions are the most common types of generalized seizures; they are characterized by loss of consciousness associated with apnea and violent contractions of the musculature of the trunk and extremities. (Answer: C—Complex partial seizure)

55. A 46-year-old diabetic man is started on insulin therapy for poorly controlled diabetes. After 2 weeks, he has a generalized tonic-clonic seizure and is brought to the emergency department by the emergency medical service. He was found in the field to have a blood sugar level of 25 mg/dl. He received a dose of 50% dextrose in water (D50W) and lorazepam, which resulted in resolution of the seizure. When you examine the patient in the emergency department, he still has mild confusion. His physical examination shows a tongue laceration; otherwise, the examination is normal. An imaging study of the brain is obtained, and the results are normal. Over the next several hours, the patient regains normal mental function.

What is the best step to take next in the treatment of this patient?
A. Refer the patient to a neurologist
B. Reevaluate the management of his diabetes
C. Obtain a sleep-deprived electroencephalogram
D. Start an antiepileptic drug

Key Concept/Objective: To understand the differential diagnosis of epilepsy

Although sudden alterations in neurologic function are characteristic of seizures, sudden alterations also occur when intracranial structures are deprived of glucose or oxygen. Although a simple syncope usually does not cause any motor activity, prolonged interruption of cerebral perfusion may cause convulsive movements or even tonic-clonic seizures. Sensory or motor dysfunction may be caused by transient ischemia. Seizures may occur with embolic strokes. Metabolic disease, particularly disease related to glucose metabolism that requires regulation of blood sugar concentration with insulin therapy, may be associated with episodes of hypoglycemia. Both focal and generalized seizures can be manifestations of hypoglycemia or of hyperosmolar states. Lumbar puncture should be performed if the patient is febrile or has altered cognitive function. To prevent complications from lumbar puncture, the clinician must first exclude the presence of an intracranial mass or increased intracranial pressure. This patient developed symptomatic hypoglycemia with a secondary tonic-clonic seizure, which resolved with correction of the hypoglycemia. No further treatment and evaluation for a seizure disorder is warranted. However, reevaluation of the management of the diabetes is of paramount importance. (Answer: B—Reevaluate the management of his diabetes)

56. A 14-year-old boy with a history of juvenile myoclonic epilepsy is seen for recurrent seizures. His disease was very well controlled in the past with valproate, and he was seizure-free for 18 months. During the past 2 months, the patient has had eight seizures, which the patient’s father describes as being different from his usual myoclonic jerks and generalized tonic-clonic seizures. These seizures are different each time; they last from 20 to 45 minutes. His physical examination is unremarkable.

On the basis of this patient’s clinical picture, what would be the most likely cause of these seizures?
A. Progression of juvenile myoclonic epilepsy
B. Intracranial mass
C. Valproate toxicity
D. Nonepileptic seizure

Key Concept/Objective: To recognize the clinical picture of nonepileptic seizures

Approximately 20% of patients admitted to epilepsy monitoring units for diagnostic evaluation have episodic behavioral alterations that are not caused by physiologic dysfunction of the brain. In the past, these alterations were called pseudoseizures; currently, the preferred term for such seizures is nonepileptic seizures. Use of this term tends to help the patients understand their problem and facilitates referral for behavioral therapy. An important clue to the diagnosis of nonepileptic seizures is that they are periodic events that tend not to be stereotyped. Both patients and observers report varied behaviors with each event. Another clue is the prolonged duration. Nonepileptic seizures may last 30 minutes to several hours—longer than typical seizures. Patients with both nonepileptic seizures and epilepsy pose a challenging problem; this combination is occasionally found in patients undergoing assessment in epilepsy monitoring units. Treatment of nonepileptic seizures requires behavioral intervention. If both disorders are found, treatment of epilepsy needs to be continued in parallel with behavioral therapy. (Answer: D—Nonepileptic seizure)

57. A 44-year-old woman is admitted to the hospital with pneumonia. She has a medical history of epilepsy, for which she has been receiving phenytoin for the past 10 years. Her last seizure occurred 14 months ago. She reports having fever, cough, and shortness of breath, but she denies having any neurologic symptoms. Her physical examination shows increased breath sounds at the right base, consistent with pneumonia. Her neurologic examination, including gait, is normal. The patient is started on antibiotics. In the emergency department, the patient’s phenytoin level was assessed; results show the phenytoin level to be elevated at 25 µg/ml (normal, 10 to 20 µg/ml).

Which of the following would be the most appropriate way to address this patient’s elevated phenytoin level?
A. Continue the same regimen of phenytoin for now
B. Hold the next dose of phenytoin, then restart the previous regimen of phenytoin
C. Hold phenytoin, assess the patient’s phenytoin levels daily until they are subtherapeutic, then restart phenytoin
D. Hold phenytoin, then restart before discharge and follow the patient’s phenytoin levels periodically to make sure they are therapeutic

Key Concept/Objective: To recognize the general principles of epilepsy drug therapy

Antiepileptic drug (AED) treatment should be directed at both controlling seizures and, when possible, correcting the underlying disease or disorder. AEDs may be used only briefly, if at all, in patients who have had a single seizure or a few seizures resulting from a transient disorder. Patients who have recurrent seizures should be treated with AEDs. Treatment with AEDs should follow certain basic principles. Therapy should be started with a single agent. Seizure control should be achieved, if possible, by increasing the dosage of this agent. If seizure control cannot be achieved with the first medication, an alternative agent should be considered. Monotherapy can control seizures in about 60% of the patients with newly diagnosed epilepsy. The use of two or more AEDs should be avoided if possible, but drug combinations may be useful when monotherapy fails. Drug selection should be guided by the patient’s seizure type and epilepsy syndrome classification in concert with the mechanisms of action and side effects. Changes in dosage should be guided by the patient’s clinical response rather than by drug levels; inadequate seizure control indicates the need for increasing the dose, and toxicity indicates the need to lower the dosage. Monitoring of levels is usually not necessary for patients who tolerate their medication well and have adequate seizure control. In some circumstances, the monitoring of drug levels may be useful in determining prescription compliance or to explain changes in seizure control or drug toxicity. This patient’s seizures are adequately controlled, and there are no clinical symptoms or signs of toxicity; therefore, changes in the dosage are not indicated, and phenytoin levels should not be followed. (Answer: A—Continue the same regimen of phenytoin for now)

For more information, see Wheless JW: 11 Neurology: XII Epilepsy. ACP Medicine Online (www.acpmedicine.com). Dale DC, Federman DD, Eds. WebMD Inc., New York, May 2003

Disorders of Sleep

58. A 48-year-old man presents to your clinic complaining of excessive daytime somnolence. His symptoms started 2 years ago. They have slowly progressed to the point where he falls asleep frequently throughout the day. The patient also reports having early morning headaches. He has tried taking naps during the day, without relief of his somnolence. His physical examination is significant for obesity and hypertension.

Which of the following tests would provide the most helpful information for the diagnosis and treatment of this patient?
A. Overnight polysomnography (PSG)
B. Multiple Sleep Latency Test (MSLT)
C. Actigraphy
D. Magnetic resonance imaging of the brain

Key Concept/Objective: To understand the tests used to evaluate sleep disorders

The two most important laboratory tests for sleep disorders are the all-night PSG study and the MSLT. This patient’s presentation is consistent with obstructive sleep apnea syndrome (OSAS); the best diagnostic test for OSAS is PSG, because it provides both diagnostic and therapeutic information. The all-night PSG study simultaneously records several physiologic variables by use of electroencephalography (EEG), electromyography (EMG), electrooculography (EOG), electrocardiography, airflow at the nose and mouth, respiratory effort, and oxygen saturation. Such studies are important in confirming a diagnosis of excessive daytime somnolence (EDS) or OSAS, and they also document the severity of sleep apnea, hypoxemia, and sleep fragmentation. Overnight PSG determines the optimal pressure for continuous positive airway pressure (CPAP)—a treatment for OSAS—and is also helpful for supporting the diagnosis of narcolepsy and the parasomnias. Overnight PSG with simultaneous video recording can confirm rapid eye movement (REM) sleep behavior disorder and is particularly useful for the documentation of unusual movements and behavior during nighttime sleep in patients with parasomnias and nocturnal seizures. The MSLT is essential in documenting pathologic sleepiness (sleep-onset latency of less than 5 minutes) and in diagnosing narcolepsy; the presence of two sleep-onset REMs with four or five naps and pathologic sleepiness strongly suggest narcolepsy. Another important laboratory test for assessing sleep disorders is actigraphy. This technique utilizes an actigraph worn on the wrist or ankle to record acceleration or deceleration of body movements, which indirectly indicates sleep-wakefulness. Actigraphy employed for days or weeks is a useful laboratory test in patients with insomnia and circadian rhythm sleep disorders, as well as in some patients with prolonged daytime sleepiness. Actigraphy is not the test of choice for patients with suspected OSAS. Magnetic resonance imaging studies and other neuroimaging techniques should be performed to exclude structural neurologic lesions if indicated; MRI will not make a diagnosis of a sleep disorder, but it can detect lesions associated with sleep disorders. (Answer: A—Overnight polysomnography [PSG])

59. A 19-year-old man is being evaluated for excessive somnolence. His symptoms appeared a few months ago, when he started to experience an irresistible desire to sleep during the day; he would then sleep for 20 or 30 minutes. He also reports having vivid hallucinations when falling asleep at night. His physical examination is unremarkable.

Which of the following is likely to be found in this patient?
A. Delta waves that occupy more than 50% of sleep during a daytime nap
B. Excess of hypocretin in the hypothalamus
C. Seizure activity on electroencephalography
D. Hypocretin deficiency

Key Concept/Objective: To understand the pathophysiology of narcolepsy

This patient likely has narcolepsy. Narcolepsy is a disorder of unknown etiology. A strong association exists between narcolepsy and the presence of the DR-15 subtype of DR2 and the DQB1*0602 subtype of DQw1 haplotypes. The most exciting recent development in our understanding of narcolepsy is the documentation of an abnormality in the hypocretin neurons in the lateral hypothalamus. Human narcolepsy-cataplexy can be considered a hypocretin (orexin) deficiency syndrome. Four lines of evidence can be cited in support of hypocretin abnormality in narcolepsy: (1) induction of narcolepsy-like symptoms after mutation of the hypocretin receptor 2 gene in dogs and after preprohypocretin knockout in mice; (2) decreased hypocretin 1 levels in the cerebrospinal fluid of narcolepsy-cataplexy patients; (3) postmortem documentation of a decrease in the number of hypocretin neurons in narcoleptic brains; and (4) preprohypocretin gene mutation in a child with severe narcolepsy associated with a generalized absence of hypocretin peptides in the brain. In narcolepsy, REM sleep is seen during attacks. Delta waves that occupy more than
50% of sleep are seen in stage IV non-REM sleep. (Answer: D—Hypocretin deficiency)

60. A 56-year-old woman is being evaluated for insomnia of recent onset. For the past 2 months, she has been waking up at 3:00 A.M. every morning and has been having difficulty returning to sleep. She denies having difficulty falling asleep at bedtime. Review of systems is significant for poor appetite and weight loss. The physical examination is unremarkable.

Which of the following interventions is the most appropriate in the treatment of this patient?
A. Start a benzodiazepine at bedtime
B. Establish sleep-hygiene measures
C. Start a sedative-antidepressant
D. Start melatonin at bedtime

Key Concept/Objective: To understand the treatment of insomnia

Insomnia may occur at any age. The patient may complain of difficulty initiating or maintaining sleep or of awakening early in the morning and being unable to go back to sleep. Insomnia may be associated with a variety of medical, psychiatric, and neurologic illnesses or may be drug or alcohol induced. Insomnia is most commonly caused by psychiatric or psychophysiologic disorders, depression and anxiety being among the most important. Early morning awakening is a characteristic finding in depression. In some cases, no cause of the insomnia is found; this disorder is termed idiopathic, or primary, insomnia and is a lifelong condition. For transient insomnia or insomnia of short duration, treatment with sedative-hypnotics (e.g., zolpidem) or shortor intermediate-acting benzodiazepines for a few nights to a few weeks is appropriate. Hypnotic medications should not be used for chronic insomnia. The best treatment for patients with chronic insomnia consists of a combination of sleep-hygiene measures (e.g., setting fixed times for retiring and awakening; avoiding caffeinated beverages, tobacco, and alcohol before retiring; and regular exercise, preferably undertaken 4 to 6 hours before bedtime), stimulus-control therapy, sleep restriction, relaxation training, and other psychological treatments. Sedative-antidepressants should be used for insomnia associated with depression. Melatonin has been found to be useful in some persons with jet lag and shift-work sleep disorders and in patients with non–24-hour circadian rhythm disorders. (Answer: C—Start a sedative-antidepressant)

61. A 53-year-old man who is otherwise healthy presents with excessive daytime somnolence. The patient has been increasingly fatigued during the day for the past several years and is now experiencing an overwhelming need for a nap during the day. He typically goes to bed around 9:45 P.M. and awakens at 6:00
A.M. He does not feel refreshed upon awakening in the morning. He takes no medications and does not use alcohol. He sleeps alone and has no unusual awakenings during the night. He has been told that he snores at night. He occasionally experiences early-morning headache. His body mass index is 24. The rest of his exam is normal.

What should be the next step in the management of this patient’s condition?
A. Referral to an ENT specialist for uvulopalatopharyngoplasty
B. Therapeutic trial of modafinil
C. Therapeutic trial of clonazepam
D. All-night polysomnography
E. Weight loss

Key Concept/Objective: To understand the treatment approaches for a patient suspected of having obstructive sleep apnea syndrome (OSAS)

This patient has excessive daytime somnolence. This may result from decreased sleep quantity, OSAS, narcolepsy, or sleep disturbance caused by restless leg syndrome. Excessive daytime somnolence caused by OSAS is commonly associated with an airway obstruction. Respiration may be disturbed during normal sleep because of an increase in upper airway resistance. This increase occurs as a result of the loss of muscle tone in the upper airways during sleep. Ventilatory responses are also decreased during sleep. Most healthy people have brief periods of apnea. Although excessive body weight is a risk factor for OSAS, approximately 30% of patients who have OSAS have normal body weight. For the decrease in respiration to be considered pathologic, the sleep apnea or hypopnea must last for at least 10 seconds, and these episodes must occur at a rate of at least five times per hour of sleep. The diagnosis is suggested by the patient’s history and is confirmed by sleep study. (Answer: D—All-night polysomnography)

62. A 52-year-old man presents with fatigue that has been increasing for the past 9 months. He describes an inability to stay awake during the midafternoon hours. He has symptoms of mild benign prostatic hyperplasia with 2 awakenings during the night to urinate. He is generally able to fall back asleep at those times, but he experiences early-morning awakenings with some difficulty in returning to sleep at that time. He has a history of chronic hepatitis B infection but has had no signs of cirrhosis or liver dysfunction for the past 10 years. He has a history of alcohol dependence, which has been in remission for the past 12 years. He consumes three cups of caffeinated products during the morning hours. He is an architect and professor at a community college and works long hours in his own consulting business. He describes his mood as average but has noted a decreased interest in his hobbies. His examination is unremarkable.

What should be the next step in managing this patient’s fatigue?
A. Liver biopsy
B. Michigan Alcoholism Screening Test
C. An evening dose of an alpha1-adrenergic blocking agent
D. A trial of an antidepressant
E. A trial of a benzodiazepine

Key Concept/Objective: To understand that depression is a common cause of insomnia

There are several potential causes of this patient’s insomnia. First, although the urinary symptoms he is experiencing may interfere with sustained and refreshing sleep, he relates no difficulty in returning to sleep after urinating. Second, alcohol use is known to be a contributing factor in decreasing sleep effectiveness. Although this remains a possibility in this case, the 12-year history of abstinence should be taken at face value unless other data emerge that suggest alcohol relapse. Chronic hepatitis B infection can be a factor in producing fatigue, but more evidence of progressive disease would be needed to implicate this as a cause of his problems. Excessive caffeine use may be a contributing factor here, but caffeine typically impedes sleep initiation rather than causes early-morning awakenings. The most likely explanation for this patient’s current fatigue is masked depression, in which mood disturbance is not a prominent feature but anhedonia and insomnia are. The use of benzodiazepines generally should be avoided in patients with a history of alcohol dependence. (Answer: D—A trial of an antidepressant)

63. A 12-year-old boy is seen for evaluation of several episodes of confusion and inappropriate behavior in the middle of the night. The patient has no symptoms during the day and is able to return to sleep after these nocturnal episodes. He is healthy, takes no medications, and is progressing well in school; family support is strong. His exam is unremarkable.

Which of the following is the most likely explanation for this patient’s problem?
A. Partial arousal disorder
B. Somnambulism
C. Pavor nocturnus
D. Psychological stress
E. Drug withdrawal

Key Concept/Objective: To understand the classification of partial arousal disorders

Partial arousal disorders include confusional arousals, sleepwalking (somnambulism), and sleep terrors (pavor nocturnus). These conditions are a subset of the parasomnias: disorders that occur during the sleep-wake transitions and during partial arousals. Parasomnias are characterized by abnormal movements or behaviors that intrude into sleep without disturbing sleep architecture. An overnight sleep study with simultaneous video recording can confirm unusual movements or behavior during nighttime sleep in patients with parasomnias. This patient has confusional arousals, which are characterized clinically by mild automatic and inappropriate behavior and confusion; they occur during slow-wave sleep. Most partial arousal disorders are benign. Sleepwalking is common in children between 5 and 12 years of age; most episodes last 10 minutes or less. There is a high probability that patients with sleepwalking have a family history of sleepwalking. Many patients with sleep terrors also have sleepwalking episodes. (Answer: A—Partial arousal disorder)

64. A 17-year-old woman presents to her primary care physician complaining of excessive tiredness. This has become increasingly serious for her over the past year. She has had difficulty staying awake in class. She falls asleep easily at night and generally is in bed at 10:00 P.M. and awakens at 6:30 A.M. She is an aboveaverage student, uses no recreational drugs or alcohol, and is not sexually active. She denies having depressive symptoms. Her family history is unremarkable. Her body weight, complete blood count, and TSH level are normal.

What should be the next step in this patient’s workup?
A. Multiple Sleep Latency Test
B. Trial of antidepressants
C. Urine drug screen
D. Serum cortisol
E. Assessment of serum free T3 level

Key Concept/Objective: To understand the presentation of narcolepsy without cataplexy

Narcoplepsy is characterized by “sleep attacks” and cataplexy. The narcoleptic attacks begin between ages 15 and 25 years, and the prevalence of this disorder is higher in patients with a family history of narcolepsy. The cause is unknown. The manifestations include an irresistible urge to fall asleep at inappropriate times; attacks last less than 30 minutes. Most patients experience cataplexy (a transient loss of muscle tone) after several years of narcolepsy. Narcolepsy becomes less severe with age. Multiple sleep-onset latency testing can be useful in diagnosing narcolepsy (sleep-onset latency refers to time to unconsciousness after attempting sleep; normal is about 5 minutes). A finding of sleep-onset latency of less than five minutes with REM sleep occurring in two out of five nap studies supports the diagnosis of narcolepsy. (Answer: A—Multiple Sleep Latency Test)

65. A 52-year-old man presents with increasing fatigue of 2 years’ duration. He has a history of mild hypertension without end-organ changes, which is being treated with a diuretic. He occasionally awakens with headaches. He naps during the day for about 30 minutes when he can and almost always falls asleep while watching the news after work. He uses no alcohol and does not smoke. His exam is remarkable for a body mass index of 29. His blood pressure is 142/88 mm Hg. The rest of his exam is normal. Serum electrolytes are remarkable for a potassium level of 3.5, and TSH is normal.

Which of the following is the most likely cause of this patient’s fatigue?
A. Hypokalemia
B. Deconditioning
C. Medication side effect
D. Sleep disturbance
E. Cerebrovascular disease

Key Concept/Objective: To understand the presenting complaints of patients with excessive daytime somnolence

Patients with excessive daytime somnolence (EDS) commonly awaken in the morning not feeling refreshed. Insufficient sleep is among the most common causes of EDS. Others include obstructive and central sleep apnea, narcolepsy, and periodic limb movements. These patients commonly have a decreased sleep-onset latency, fatigue upon awakening, and urges to sleep during the day. History-taking should be directed at sleep patterns, drug and alcohol use, and psychiatric illness. If an obvious cause of sleep disturbance is not found during the clinical exam, a sleep study can help identify such causes as obstructive sleep apnea, restless leg syndrome, and periodic limb movement in sleep. (Answer: D—Sleep disturbance)

For more information, see Chokroverty S: 11 Neurology: XIII Disorders of Sleep. ACP Medicine Online (www.acpmedicine.com). Dale DC, Federman DD, Eds. WebMD Inc., New York, October 2003

Pain

66. Complaints of pain are among the most common reasons for patients to visit a health care professional. New pain complaints account for close to 40 million physician visits annually in the United States.

Which of the following statements regarding pain is false?
A. Chronic pain, in contrast to acute pain, does not warn the patient of bodily injury and serves no useful function
B. Neuropathic pain is caused by injury to the peripheral nervous system or CNS and can occur chronically without ongoing damage
C. Between one third and one half of cancer patients report pain that cannot be controlled with analgesics
D. Treatment of chronic pain should not be undertaken unless physical examination reveals demonstrable pathology, such as neurologic changes or signs of duress (e.g., tachycardia)
E. Inquiries about psychosocial and financial factors related to pain are an important part of an initial pain evaluation

Key Concept/Objective: To understand that chronic pain is common and to know the basic tenets of the management of chronic pain

Pain is a subjective experience, and its expression is unique to each patient. Often there is little objective evidence with which to assess the source or intensity of pain. Thus, one of the most important aspects of the patient-physician relationship regarding the treatment of chronic pain is trust: the physician is obligated to rely on the patient’s self-reports of pain; to do otherwise may be unethical. Pain is a complex process that involves biologic and psychosocial factors. It can be classified as somatic (involving activation of nociceptors in cutaneous and deep musculoskeletal tissues), visceral (resulting from abnormal forces on thoracic, abdominal, and pelvic viscera), and neuropathic (resulting from injury to the peripheral nervous system or the CNS). Pain complaints are extremely common in patients with chronic disease, such as cancer and AIDS; over three fourths of such patients report pain symptoms. Unfortunately, a large percentage of patients with terminal cancer have pain that is inadequately controlled. A detailed financial and psychosocial history is of paramount importance because of the multifactorial nature of pain. (Answer: D— Treatment of chronic pain should not be undertaken unless physical examination reveals demonstrable pathology, such as neurologic changes or signs of duress [e.g., tachycardia])

67. A 50-year-old diabetic woman has diabetic nephropathy and neuropathy that involves her lower extremities. She complains of paresthesias and chronic lancinating pains in the feet. She has received treatment with several nonsteroidal anti-inflammatory drugs (NSAIDs), and for the past 6 months she has been taking a combination of acetaminophen and codeine. Despite this, she has persistent symptoms. Her pain has limited her ability to perform her job, which requires spending long periods of time on her feet.

Which of the following is the most appropriate option for treating this patient’s chronic pain?
A. Substitution of oral meperidine for her current analgesic regimen
B. Addition of an adjuvant analgesic such as gabapentin or a tricyclic antidepressant to her regimen
C. Immediate discontinuance of opioid medication and referral to physical therapy
D. The addition of high-dose ibuprofen three times daily to her current regimen

Key Concept/Objective: To understand the use of adjuvant medications for the treatment of neuropathic pain

Neuropathic pain is common in patients with diabetic neuropathy and in those who have had shingles (postherpetic neuralgia). The pain is often described as a constant, dull ache; superimposed episodes of burning or electric shock–like sensations are common. Several studies have demonstrated the efficacy of both tricyclic antidepressants and gabapentin in the treatment of neuropathic pain related to these conditions. These medications are often used as first-line agents in the treatment of neuropathy and are also useful as adjuncts to opioids in this setting. Oral meperidine is generally not recommended for chronic use because of the potential for the buildup of toxic metabolites. Chronic narcotic medications should not be abruptly discontinued because of the potential for withdrawal symptoms. The administration of high-dose NSAIDs would not be the appropriate step to take next in the treatment of this patient with nephropathy, given their effects on renal blood flow. (Answer: B—Addition of an adjuvant analgesic such as gabapentin or a tricyclic antidepressant to her regimen)

68. You are caring for a 67-year-old man with widely metastatic small cell carcinoma of the lung who was diagnosed 4 months ago. He suffers from chronic pain secondary to bony metastases. His pain regimen includes sustained-release morphine, which he takes twice daily, and fast-acting morphine for breakthrough episodes of pain. Your nurse reports that the patient has called the office to request an increase in his pain medicines. He feels that they are not as effective as they were previously. His wife reports that he is afraid to take additional doses beyond what is prescribed for fear of being “hooked on them.”

Which of the following is the patient experiencing with regard to his pain medications?
A. Addiction
B. Pseudoaddiction
C. Tolerance
D. Substance abuse
E. Drug resistance

Key Concept/Objective: To understand the differences between addiction and physical dependence

This patient is experiencing the expected physiologic results of long-term opioid use: physical dependence and tolerance. Physical dependence is a state of adaptation that is manifested by specific withdrawal symptoms that occur with abrupt discontinuance of the drug. Tolerance is a physiologic state resulting from regular use of the drug. For patients experiencing tolerance, an increased dose is needed to produce the same effect as the original dose, or a reduced effect is observed with a constant dose. Addiction is a neurobehavioral syndrome influenced by genetic and environmental factors. It results in psychological dependence on the use of substances for their psychic effects. It is characterized by compulsive use despite harm and impaired control over drug use. Pseudoaddiction is a pattern of behavior in which a patient who is receiving inadequate pain medication seeks drugs for pain. This drug-seeking behavior can be mistaken for addiction. (Answer: C—Tolerance)

69. A 35-year-old patient comes to your office for a follow-up visit after experiencing a clavicle fracture. He has a history of cerebral palsy with moderate cognitive dysfunction and a seizure disorder. He lives in a group home. One week ago, he had a generalized tonic-clonic seizure that resulted in a fall, at which time he injured his right clavicle. His clavicle injury is being treated conservatively with a sling. His caretakers have brought him to clinic with reports that he has become more withdrawn and is eating and sleeping poorly. Although communication with the patient regarding his specific symptoms is difficult, there is concern that he may be in significant pain. Using a face pain-rating scale, you are able to elicit a complaint of pain from the patient that rates 6 on a scale of 10.

Which of the following is the most appropriate pharmacologic intervention for treatment of this patient’s pain?
A. Tramadol, 50 mg p.o. every 6 hours as needed
B. Amitriptyline, 25 mg p.o. at bedtime, with titration to 100 mg over the next 2 weeks
C. Combination acetaminophen 300 mg/codeine 30 mg, 1 to 2 tablets every 4 to 6 hours as needed, plus a stool softener
D. Meperidine, 50 mg I.M. in clinic, followed by 50 mg p.o. every 4 hours as needed, plus a stool softener
E. Prednisone at an initial dosage of 60 mg a day, tapering to discontinuance over the next 2 weeks

Key Concept/Objective: To understand which analgesic medications can lower seizure threshold

The assessment and treatment of pain in patients with cognitive impairment can be challenging. Treatment of any patient must take into account any comorbid conditions, and pharmacologic therapy must be initiated carefully, with attention given to possible adverse effects. In this patient with a known seizure disorder, a combination of acetaminophen and codeine is a safe choice for short-term treatment of pain. Tramadol, a nonnarcotic analgesic that binds to mu opiate receptors in the CNS and causes inhibition of ascending pain pathways, is contraindicated in this patient because it tends to make seizures worse. Similarly, tricyclic antidepressants and the opioid analgesic meperidine can induce seizures and thus would not be the best initial choice for this patient. Although corticosteroids are potent anti-inflammatories that are useful adjuncts in certain conditions, their use here would be unlikely to give symptomatic relief. (Answer: C—Combination acetaminophen 300 mg/codeine 30 mg, 1 to 2 tablets every 4 to 6 hours as needed, plus a stool softener)

For more information, see Galer B, Gammaitoni A, Alvarez NA: 11 Neurology: XIV Pain. ACP Medicine Online (www.acpmedicine.com). Dale DC, Federman DD, Eds. WebMD Inc., New York, May 2002

Parkinson Disease and Other Movement Disorders

70. A 67-year-old woman comes to your office accompanied by her family. She has a history of multiple falls, which have been increasing over the past 6 months. She says that she feels unsteady almost all the time, is frequently light-headed, and has difficulty walking. On examination, she has bradykinesia, mild cogwheeling of both upper extremities, a blood pressure drop of 25 mm Hg on standing with no change in pulse, and an ataxic gait.

Which of the following is the most likely diagnosis for this patient?
A. Parkinson disease
B. Wilson disease
C. Multiple systems atrophy
D. Huntington disease
E. Progressive supranuclear palsy

Key Concept/Objective: To be able to recognize the symptoms of different parkinsonian disorders

Bradykinesia could occur in patients with Parkinson disease, multiple systems atrophy, or progressive supranuclear palsy. Parkinson disease is sometimes accompanied by autonomic insufficiency in its later stages, but this patient presents with only mild motor symptoms. The combination of parkinsonism, autonomic insufficiency, and ataxia is strongly suggestive of multiple systems atrophy. (Answer: C—Multiple systems atrophy)

71. A 35-year-old man is referred to your clinic for evaluation of early-onset Parkinson disease. His symptoms began approximately 2 years ago with tremor and difficulty speaking. These symptoms have progressed to the degree that he has become severely depressed and unable to work. His family history is remarkable for mental illness and alcoholism, and his maternal grandfather had cirrhosis of the liver. There is no history of Parkinson disease in his family. On examination, the patient has a mild resting tremor and uneven gait but no bradykinesia. As you speak with him, he is quite dysarthric and occasionally manifests writhing facial and neck movements. When asked if these are voluntary, he becomes angry, then tearful, and then says that God is punishing him with these movements.

Which of the following is the most appropriate step to take next in the management of this patient?
A. MRI of the brain
B. Trial of therapy with carbidopa-levodopa
C. Trial of therapy with risperidone
D. Measurement of 24-hour urinary copper excretion
E. Trial of therapy with sertraline

Key Concept/Objective: To know the symptoms of Wilson disease

This young patient presents with tremor, prominent dysarthria, and facial dystonias. The latter two symptoms are atypical of Parkinson disease and should lead to consideration of other possibilities. The psychiatric symptoms of irritability, depression, and delusions of reference are suggestive of Wilson disease, for which a 24-hour urinary copper excretion test would be diagnostic. Initial therapy consists of antiabsorptive therapy followed by chelation with penicillamine. Unfortunately, neurologic symptoms often do not improve with chelation. (Answer: D—Measurement of 24-hour urinary copper excretion)

72. A 68-year-old man presents to your office for evaluation of tremor. He has had this problem for 3 years and it is beginning to affect his work. He has difficulty writing and has begun to notice wavering in his voice when dictating letters (now his preferred method of correspondence). He denies having any history of regular alcohol consumption but keeps a bottle of scotch in his desk because it “steadies his nerves” before important meetings. On examination, he has a relatively fast tremor of the right hand that is enhanced significantly with finger-to-nose testing. His voice has a shaky, quivering quality. His gait is normal, but his handwriting is very difficult to read because of shaking when he writes.

Which of the following medications is most likely to help this patient?

A. Carbidopa-levodopa
B. Valproic acid
C. Selegiline
D. Amitriptyline
E. Propranolol

Key Concept/Objective: To be able to distinguish the clinical presentation and treatment of essential tremor from those of other movement disorders

This patient’s fast tremor that increases with intention and involves his voice is most likely caused by essential tremor. Parkinson symptoms typically decrease with intention, and although it can involve facial and jaw muscles, the disease usually spares phonation. Alcohol consumption often briefly suppresses symptoms of essential tremor, and beta blockers can prove to be helpful in long-term therapy. Essential tremor does not typically respond to antiparkinsonian agents, and tricyclic antidepressants or valproate can worsen the problem. (Answer: E—Propranolol)

73. A 27-year-old patient of yours comes in because her 48-year-old father has been diagnosed with
Huntington disease. She wants to know what this means for her risk of getting the disease.

What is the best thing to tell this patient?
A. She is a carrier of the gene for Huntington disease, but she is unlikely to get the disease herself unless it runs in her mother’s family
B. There is a 50% chance that she has inherited the gene for Huntington disease, but if she has, she is unlikely to show symptoms until she is in her 50s or 60s
C. There is a 50% chance that she has inherited the gene for Huntington disease, but fewer than half of the people with the gene develop the disease, so her odds are not too bad
D. There is a 50% chance that she has inherited the gene for Huntington disease; if she has, she is likely to show symptoms at a younger age than did her father
E. She may have inherited the gene for Huntington disease, but it usually only manifests in men because it is on the X chromosome

Key Concept/Objective: To understand the genetics of Huntington disease and the implications for families of affected patients

Huntington disease is an autosomal dominant disorder that manifests anticipation (i.e., a tendency toward earlier onset in subsequent generations). The genotype manifests when an area of glutamine (CAG) repeats on chromosome 4 exceeds 40 repetitions of the CAG codon. These repeats tend to increase in subsequent generations, a phenomenon that correlates with onset of the disease. It is a disease characterized by very high penetrance: almost all patients with the genotype develop Huntington disease. For this reason, genetic testing of asymptomatic family members is an ethically complex and difficult proposition. With regard to the alternative answers to this question, choice A characterizes autosomal recessive inheritance; choice B characterizes autosomal dominant inheritance with negative anticipation; choice C characterizes autosomal dominant inheritance with low penetrance; and choice E characterizes X-linked recessive inheritance. (Answer: D—There is a 50% chance that she has inherited the gene for Huntington disease; if she has, she is likely to show symptoms at a younger age than did her father)

For more information, see Juncos JL, DeLong MR: 11 Neurology: XV Parkinson Disease and Other Movement Disorders. ACP Medicine Online (www.acpmedicine.com). Dale DC, Federman DD, Eds. WebMD Inc., New York, April 2005

Acute Viral Central Nervous System Diseases

74. A 26-year-old elementary schoolteacher presents to the emergency department for evaluation of headache and neck stiffness. Vital signs are as follows: temperature, 100.8° F (38.2° C); heart rate, 92 mm Hg; blood pressure, 124/78 mm Hg; and respiratory rate, 14 breaths/min. On physical examination, the patient is noted to have an erythematous, maculopapular rash. The remainder of the physical examination, including neurologic examination, is unremarkable. A lumbar puncture is performed to evaluate for meningitis.

Which of the following cerebrospinal fluid findings is NOT characteristic of viral meningitis?
A. A protein level of 245 mg/dl
B. A white blood cell (WBC) count of 80 cells/mm3
C. A glucose level of 60 mg/dl
D. Clear cerebrospinal fluid (CSF)

Key Concept/Objective: To know the typical CSF findings of viral meningitis

Careful examination of the CSF is the mainstay of diagnosis of viral meningitis or encephalitis. Characteristically, the CSF is clear and features a predominantly mononuclear pleocytosis and normal glucose content. Initially, the CSF may contain polymorphonuclear leukocytes. The CSF cell count is usually below 100 cells/mm3; it may be higher with enteroviral infections, however, and the CSF may contain thousands of mononuclear cells after mumps and lymphocytic choriomeningitis (LCM) virus infections. The CSF protein concentration is usually normal or mildly elevated (not markedly elevated, as seen in this patient). Bacteria are not found on Gram stain, and CSF cultures are sterile. A mild depression in the CSF glucose content develops in approximately one third of patients with meningitis caused by mumps or LCM virus; this drop in CSF glucose level occurs less often after enterovirus infection. In rare instances, the CSF glucose content is depressed in patients with aseptic meningitis caused by herpes simplex virus (HSV) or varicella-zoster virus (VZV). (Answer: A—A protein level of 245 mg/dl)

75. A 32-year-old man presents to the emergency department with fever, headache, and neck stiffness.
Lumbar puncture reveals clear CSF with a marked lymphocytic pleocytosis, a slightly elevated protein level, and a normal glucose level. Gram stain and India ink did not reveal any organisms on culture. A presumptive diagnosis of viral meningitis is made on the basis of CSF findings. On reviewing the patient’s medical records, you discover that he has been evaluated for similar symptoms on multiple occasions in the past. Lumbar punctures performed at these visits have revealed CSF findings similar to those observed on this visit.

Which of the following viruses is most likely responsible for this patient’s recurrent episodes of viral meningitis?
A. VZV
B. HSV-1
C. Echovirus
D. HSV-2

Key Concept/Objective: To know that HSV-2 is the primary cause of benign recurrent lymphocytic meningitis

HSV-2 aseptic meningitis is the main neurologic complication of HSV-2 infection. HSV-2 causes genital herpes. In the United States, HSV-2 is the third most common cause of aseptic meningitis, accounting for approximately 5% of all cases. Unlike viral meningitides that have a seasonal association, HSV-2 meningitis occurs at any time of year. The typical symptoms and signs are headache, fever, stiff neck, and a marked lymphocytic pleocytosis in the CSF. Meningitis may be preceded by genital or pelvic pain, and the astute clinician who suspects HSV-2 meningitis will ask about recent symptoms of pelvic inflammatory disease or associated penile or scrotal pain. The workup for suspected HSV-2 meningitis should include a careful search for vesicular lesions over the external genitalia and a pelvic examination for lesions in the vagina or on the cervix. Polymerase chain reaction has revealed that the primary agent causing benign recurrent lymphocytic meningitis is HSV-2. Occasionally, HSV-1 is the culprit, as evidenced by the detection by PCR of HSV-1 DNA in the CSF of patients with benign recurrent lymphocytic meningitis. HSV-2 meningitis is self-limited; treatment with acyclovir is not required. (Answer: D—HSV-2)

76. A 72-year-old man presents to a primary care clinic for the evaluation of a painful rash on the right side of his face. Physical examination reveals a vesicular eruption in the maxillary distribution of the trigeminal nerve. The patient is diagnosed with zoster and treated with famciclovir.

Which of the following neurologic sequelae is most likely to develop in this patient in the weeks or months following this episode?
A. Ramsay Hunt syndrome
B. Viral meningitis
C. Left-sided hemiparesis
D. Viral encephalitis

Key Concept/Objective: To understand the CNS sequelae of zoster infection and to recognize that unifocal VZV vasculopathy presents with focal neurologic findings

The salient feature of VZV unifocal vasculopathy is an acute focal deficit that develops weeks or months after the development of herpes zoster in a contralateral pattern of trigeminal distribution. Stroke results from a necrotizing arteritis, primarily of large cerebral arteries. One comprehensive review showed that most patients with large vessel vasculopathy were older than 60 years and that there was no sex bias. The mean onset of neurologic disease was 7 weeks, and the longest interval between the onset of herpes zoster and the onset of neurologic disease was 6 months. Transient ischemic attacks and mental symptoms were common. Twenty-five percent of patients died. The majority of patients had CSF pleocytosis, usually fewer than 100 cells/mm3 (predominantly mononuclear), oligoclonal bands, and increased CSF IgG. Besides contralateral hemiplegia, ipsilateral central retinal artery occlusion and posterior circulation involvement have been described. There is no definitive treatment for large vessel herpes zoster vasculopathy. Nevertheless, because productive virus infection is found in arteries, patients should receive intravenous acyclovir (to kill persistent replicating virus) and steroids (for their anti-inflammatory effect). Overall, the neurologic features of VZV vasculopathy are protean. Neurologic disease often occurs months after zoster and sometimes in patients who have no history of zoster rash. Magnetic resonance imaging, cerebral angiography, and examination of CSF with virologic analysis are needed to confirm the diagnosis. When VZV vasculopathy develops months after zoster, antiviral treatment is often effective. (Answer: C—Left-sided hemiparesis)

77. A 62-year-old woman presents to the emergency department for the evaluation of a seizure. The patient is lethargic and confused. Her husband reports she has epilepsy and that she has been complaining of headache for the past 3 days. He also states that the patient has been somewhat confused. On physical examination, the patient is noted to have a low-grade fever. The remainder of her vital signs are stable, and her examination is otherwise unrevealing. A head CT scan reveals a hypodense lesion in the right medial temporal region. A lumbar puncture is performed, revealing an elevated opening pressure, mononuclear pleocytosis, and moderate red blood cells (RBCs).

Which of the following viruses is most likely responsible for this patient’s clinical picture?
A. VZV
B. HSV-1
C. Cytomegalovirus (CMV)
D. HSV-2

Key Concept/Objective: To know the characteristic clinical features of HSV-1 encephalitis

Unlike most viral encephalitides, HSV-1 encephalitis is focal. HSV replication in the medial temporal lobe and orbital surface of the frontal lobe, with accompanying inflammation, produces the characteristic clinical picture. Fever, headache, lethargy, irritability, and confusion are typical symptoms. Seizures (major motor, complex partial, focal, and even absence attacks) affect approximately 40% of patients. If the dominant temporal lobe is involved, aphasia and focal motor or sensory deficits develop. The CSF is usually abnormal in HSV encephalitis. The CSF opening pressure is often elevated and may be very high if there is brain swelling and impending temporal lobe herniation. Clinicians usually perform the CSF examination in the first few days of illness, before there is significant brain swelling, to decrease the potential for herniation after lumbar puncture. The electroencephalogram (EEG) and imaging studies may demonstrate features highly suggestive of HSV encephalitis, often obviating the need for subsequent lumbar punctures. CSF pleocytosis is observed in more than 90% of patients, although its absence at initial evaluation does not rule out HSV encephalitis. The CSF cell count ranges from 4 to 755 cells/mm3, and more than 200 cells/mm3 may be present weeks after the onset of disease. The predominant cell type is mononuclear. Although RBCs are unusual in other viral encephalitides, in HSV encephalitis they are often present in the CSF, which may also be xanthochromic; this presumably reflects the hemorrhagic nature of brain lesions. Instead of attributing the presence of RBCs in CSF to a so-called traumatic tap, the astute clinician may use this finding to support the presumptive diagnosis of HSV encephalitis. The majority of patients with HSV encephalitis have elevated CSF protein and IgG indexes. In rare instances, hypoglycorrhachia occurs. Increased levels of antibody to HSV, suggestive of recent infection, may be found in serum and CSF; increased anti-HSV antibody ratios of CSF and serum may help in making the diagnosis of HSV encephalitis. Unfortunately, increased antibody titers are not usually detected until 2 weeks or longer after the onset of disease; thus, their practical value lies more in retrospective presumptive diagnosis than in identifying acute encephalitis. PCR detection of HSV-1 DNA in the CSF is both sensitive and specific and has become the gold standard in suspected cases of HSV encephalitis. Nonetheless, clinicians should be aware that PCR may be negative for HSV in the first few days of illness with HSV encephalitis. (Answer: B—HSV-1)

For more information, see Gilden DH: 11 Neurology: XVI Acute Viral Central Nervous System Diseases. ACP Medicine Online (www.acpmedicine.com). Dale DC, Federman DD, Eds. WebMD Inc., New York, March 2003

Central Nervous System Diseases Due to Slow Viruses and Prions

78. A 34-year-old man was found to be HIV seropositive 4 years ago. He refused antiretroviral therapy. He began having headaches and low-grade fevers 1 month ago. Previously, his friends noted no change in his functioning, personality, or thinking, but now, he is becoming progressively more confused and has difficulty caring for himself. On examination, the patient’s temperature is 100° F (37.8° C). He is sleepy but arouses easily. He has no nuchal rigidity. A mental status examination shows diminished cognitive ability. There are no focal motor or sensory findings. A magnetic resonance imaging study shows global cortical atrophy, no ventricular enlargement, and no focal lesions.

What is the most likely diagnosis in this case?
A. Toxoplasmosis
B. Progressive multifocal leukoencephalopathy (PML)
C. Central nervous system lymphoma
D. HIV dementia complex
E. Cryptococcal meningitis

Key Concept/Objective: To understand that because multiple CNS diseases can occur with HIV infection, HIV dementia remains a diagnosis of exclusion

Toxoplasmosis presents with fever, headache, focal exam findings, and multiple ringenhancing brain lesions on imaging studies. PML presents with focal deficits on exam without alteration in consciousness and with multiple white matter lesions on T2-weighted MRI images. CNS lymphoma presents with focal neurologic deficits, headache, and one to a few focal brain lesions on radiographic studies. HIV dementia is associated with a slower progression of personality changes, dementia, and unsteady gait. Cortical atrophy and ventricular enlargement are commonly seen on MRI, but the findings are not diagnostic. Atrophy can be seen in patients with AIDS who otherwise would exhibit only subtle findings on formal psychological testing. Cryptococcal meningitis is an opportunistic infection presenting with fever, headache with or without nuchal rigidity, and confusion. (Answer: E—Cryptococcal meningitis)

79. A 42-year-old woman from Florida prevents for evaluation. Four years ago, she developed leg numbness, constipation, and urinary incontinence developed, and she became wheelchair-bound 12 months ago. Skin examination reveals scattered red-brown maculopapular lesions. Neurologic examination reveals decreased pinprick sensation at the thoracic level, paraplegia, increased leg muscle tone, brisk leg reflexes, and Babinski signs. Serologic tests for Lyme disease, syphilis, and HIV are negative. Skin biopsy reveals lymphoid infiltrates within the epidermis. MRI reveals areas of T2-weighted increased signal intensity and atrophy in the thoracic spinal cord.

What is the most likely diagnosis for this patient?
A. HIV vacuolar myelopathy
B. Poliomyelitis
C. Human T cell lymphotropic virus type I (HTLV-I)–associated myelopathy (HAM)
D. Multiple sclerosis
E. PML

Key Concept/Objective: To understand the need to consider a retroviral cause for a myelopathy that can mimic multiple sclerosis

PML is a disease of brain white matter presenting with motor and vision findings and eventual dementia. HIV involvement of the cord does occur, but this patient has no serologic evidence for HIV infection. Poliomyelitis involves the anterior horn cells of the cord, causing a flaccid paralysis with absent reflexes rather than spastic paraparesis. Radiologic findings can be similar in HAM and multiple sclerosis. Both diseases can cause elevated IgG and oligoclonal bands in the CSF. Although HAM is reported mainly in Japan, the Caribbean region, and Central America, it does occur in the southeastern United States. HTLV-I can also cause a T cell lymphoma/leukemia, and although it is rare to have the neurologic and hematopoietic complications occur at the same time, this patient’s skin biopsy is characteristic of a cutaneous lymphoma. Serology testing should be done to confirm HTLV-I infection. (Answer: C—Human T cell lymphotropic virus type I [HTLV-I]–associated myelopathy [HAM])

80. A 54-year-old beef farmer was visiting this country on an extended visit. For a period of 2 months he experienced difficulty reading, and his relatives took him to the hospital for evaluation. On examination, the patient was agitated; he had memory impairment, and a left homonymous defect was noted on visual-field testing. He was admitted to the hospital, and 1 week later he developed ataxia and myoclonic jerks of the hands. CT scan and MRI were negative. An EEG showed triphasic wave forms. One month later, he was barely responsive, with severe generalized myoclonic jerks, and he died shortly afterward. A review of patient history revealed that five years ago, two animals in his beef herd developed mad cow disease. There is no family history of a similar neurologic disorder. He underwent no surgeries or blood transfusions.

This patient most likely had which form of transmittable spongiform encephalopathy?
A. Kuru
B. Sporadic Creutzfeldt-Jakob disease (CJD)
C. Gerstmann-Strussler-Scheinker disease (GSS)
D. Scrapie
E. Variant Creutzfeldt-Jakob disease (vCJD)

Key Concept/Objective: To be familiar with the characteristics of the variety of transmittable spongiform encephalopathies

Kuru previously was the disease associated with cannibalism in New Guinea but no longer occurs. Scrapie is the spongiform encephalopathy that occurs in sheep. GSS is a human spongiform encephalopathy, transmitted as an autosomal dominant trait, that presents with ataxia followed by dementia. Variant CJD is believed to be the accidental transmission to humans of the agent of bovine spongiform encephalopathy (mad cow disease). It differs from CJD in that it occurs in younger patients. Patients present with psychiatric disturbances; dementia develops much later. Patients with CJD lack characteristic EEG findings and have a longer survival time. This patient had classic findings of CJD: visual changes, dementia, ataxia, myoclonic jerks, a characteristic EEG study, and rapid progression to death. It was the sporadic rather than the genetic or iatrogenic form. (Answer: B— Sporadic Creutzfeldt-Jakob disease [CJD])

81. What is the pathogenetic mechanism for the disease of the patient in Question 80?
A. Secretion of neurotoxins from microglia chronically infected with a retrovirus
B. Oligodendroglial cytolysis caused by a papovavirus
C. CD8+ T cell attack on neuroglial cells minimally infected with a retrovirus
D. Misfolding of a protease-resistant protein causing plaques in the brain
E. Mutation in the matrix protein of a virus causing viral persistence

Key Concept/Objective: To be familiar with theories of pathogenesis for CNS diseases caused by slow viruses and similar agents

Spongiform encephalopathies are caused by prions: proteins that, when abnormal, precipitate as crystals, yielding plaques and spongiform degeneration. HIV dementia is thought to be caused by injury to neurons by toxins secreted from HIV-infected microglia. HTLV-I–associated myelopathy is thought to be the result of autoimmune attack by CD8+ T cells on HTLV-I–infected glial cells. Lysis of cells that produce myelin (oligodendrocytes) by reactivated JC virus, a papovavirus, is the cause of progressive multifocal leukoencephalopathy. Subacute sclerosing panencephalitis (SSPE) is a late CNS disease caused by the measles virus containing mutated protein, perhaps accounting for viral persistence in the presence of an appropriate host-humoral response. (Answer: D—Misfolding of a proteaseresistant protein causing plaques in the brain)

82. An HIV-seropositive 36-year-old woman presents with progressive right-sided weakness and right inferior quadrantanopia. MRI reveals hypointense T1-weighted lesions without gadolinium enhancement and with hyperintense T2-weighted lesions involving the left frontoparietal and occipital white matter. Biopsy shows areas of demyelination, large atypical astrocytes with bizarre nuclei, and oligodendrocytes with enlarged nuclei, some with displacement of the chromatin by an intranuclear basophilic process.

What is the most appropriate therapy for this patient at this time?
A. Ethambutol, rifabutin, and clarithromycin
B. Aggressive therapy for HIV
C. Pyrimethamine and sulfadiazine
D. Cytarabine
E. Radiation therapy

Key Concept/Objective: To understand the findings of progressive multifocal leukoencephalopathy (PML) and current thought regarding its therapy

This patient has a classic presentation and studies for PML. On MRI, the lack of contrast enhancement is typical for PML, as distinct from toxoplasmosis, lymphoma, or astrocytoma. Bizarre cells are seen on biopsy but do not indicate the presence of a malignancy. The intranuclear inclusions in the oligodendrocytes are typical for PML. There are no proven therapies of benefit in managing PML, although it is believed that partial reversal of the immunodeficient state may be responsible for transient improvement. Cytarabine, a cancer drug, was thought to be of benefit in isolated case reports, but controlled studies in AIDS patients have not borne this out. The patient does not have toxoplasmosis requiring pyrimethamine and sulfadiazine, nor does she have Mycobacterium avium complex infection requiring ethambutol, rifabutin, and clarithromycin or CNS malignancy requiring radiation. (Answer: B—Aggressive therapy for HIV)

83. A 62-year-old man weighing 70 kg presents with a 2-day history of fever, headache, increasing confusion, and a single generalized seizure. On examination, he has a temperature of 102° F (38.9° C), is drowsy and confused, has no nuchal rigidity, and has an upgoing right plantar reflex. CT scan is normal, but a T2-weighted MRI scan shows increased signal intensity in the left temporal lobe. Peripheral WBC is normal, as are the serum electrolytes. Lumbar puncture reveals an opening pressure of 250 mm H2O; clear cerebrospinal fluid with protein 100 mg/dl; glucose, 70 mg/dl, with simultaneous blood glucose
120 mg/dl; 40 WBC/mm3, predominantly mononuclear; and 850 RBC/mm3. Gram stain of the fluid shows no organisms.

Which of the following test is likely to be most helpful in confirming your clinical diagnosis at this point?
A. CSF for viral tissue culture
B. CSF for polymerase chain reaction (PCR) detection of viral DNA
C. CSF for viral antibody detection
D. EEG
E. Brain biopsy for pathology and viral tissue culture

Key Concept/Objective: To understand the tests to confirm the diagnosis of HSV-1 encephalitis

A rapid neurologic deterioration, seizure, fevers, focal examination and radiographic findings, and mononuclear CSF pleocytosis with increased RBCs are enough for a clinical diagnosis of HSV encephalitis. PCR detection of HSV-1 DNA in the CSF has become the gold standard to confirm suspected cases of HSV encephalitis. Although CSF viral cultures are performed, they have a very low yield. CSF viral antibody levels can be detected after 2 or more weeks of illness. EEG findings usually show generalized slowing early and may later develop more characteristic periodic sharp-wave and slow-wave complexes. With focal temporal lobe involvement on MRI, an EEG in this case would add little. Brain biopsy for culture and pathology was the previously recommended confirmatory test. It is not as sensitive as the PCR studies and carries greater risks. (Answer: B—CSF for polymerase chain reaction [PCR] detection of viral DNA)

84. Which of the following treatment regimens would you now begin for the patient in Question 83?
A. Acyclovir, 700 mg I.V., q. 8 hr, and maintenance I.V. fluids at a rate of 150 ml/hr
B. Acyclovir, 700 mg I.V., q. 8 hr, maintenance I.V. fluids at 150 ml/hr, and phenytoin
C. Acyclovir, 700 mg I.V., q. 8 hr, and maintenance I.V. fluids at 50 ml/hr
D. Acyclovir, 700 mg I.V., q. 8 hr, maintenance I.V. fluids at 50 ml/hr, and phenytoin
E. Ganciclovir, 350 mg I.V., q. 12 hr, and maintenance I.V. fluids at 150 ml/hr

Key Concept/Objective: To understand the treatment of HSV encephalitis

The earlier the initiation of antiviral therapy in HSV encephalitis, the better is the outcome. Treatment should not be withheld pending confirmatory results of diagnostic testing. The drug to use is acyclovir, 10 mg/kg I.V., every 8 hours, for at least 10 days. Ganciclovir is the drug to use if the patient has CMV encephalitis. Because the patient has already had a generalized seizure, he should be started on anticonvulsants, which should be continued for at least several months after the acute illness, should the patient survive. There are no controlled studies to support the use of prophylactic anticonvulsants, had the patient not had seizures. Cerebral edema is a significant concern in patients with HSV encephalitis. Fluids should be reduced to approximately 50% of usual maintenance levels. (Answer: D—Acyclovir, 700 mg I.V., q. 8 hr, maintenance I.V. fluids at 50 ml/hr, and phenytoin)

85. A 15-year-old boy lives adjacent to a swampy region of Florida. He presents with worsening stupor after 3 days of fevers, headache, nausea, and vomiting. On examination, the patient has a temperature of 101° F (38.3° C). He is disoriented with marked lethargy. There is no nuchal rigidity and no focal neurologic findings. Initial laboratory studies show a leukocytosis of 14,500 cells/mm3 and mild hyponatremia. Results of CSF analysis are as follows: 370 WBC/mm3, with 70% neutrophils; protein, 95 mg/dl; glucose, 68 mg/dl, with simultaneous serum glucose, 110 mg/dl. T2-weighted MRI images reveal increased signal intensity in the basal ganglia and thalami.

Which of the following statements regarding this patient’s disease is true?
A. The disease spreads primarily via the fecal-oral route
B. Neutrophilic CSF pleocytosis is distinctly unusual early in the course of this disease
C. Using sentinel chickens to detect viral infection is important in controlling outbreaks
E. The mortality from this disease is less than 20%
E. The disease is caused by an arenavirus

Key Concept/Objective: To be familiar with the presentation of an arboviral encephalitis and the means to detect the mosquito vectors

Rapid alteration in mental status associated with fevers should raise suspicion for viral encephalitis. Of the various types of viral encephalitides, Eastern equine encephalitis (EEE), in a recent review, was found to have distinctive basal ganglion and thalamus involvement on MRI scan. It is one of the arboviral (arthropod-borne) encephalitides and is spread by mosquito bite rather than a fecal-oral route. Neutrophilic CSF pleocytosis would not be unusual early in the course of any acute viral CNS infection and is common in EEE. The mortality is 35% to 50%. Control of outbreaks comes from monitoring and controlling the mosquito vectors. Presence of the virus in swampy habitats can be detected by recovering virus from mosquitoes or by measuring serum antibodies in wild passerine birds or caged sentinel birds (chickens). EEE is caused by an alphavirus. The arenaviruses are endemic in rodents, with lymphocytic choriomeningitis virus being most common in the United States. (Answer: C—Using sentinel chickens to detect viral infection is important in controlling outbreaks)

86. A 59-year-old woman from Missouri has a 20-year history of systemic lupus erythematosus, for which she is taking corticosteroids. One day before admission, the patient started taking ibuprofen for a flare in arthralgias. The next day, she was found obtunded and confused. She has a temperature of 100.2° F (37.9° C); she can be aroused but she cannot follow simple commands. The rest of the examination is normal. Chest x-ray, complete blood count, and urinalysis are normal. CSF analysis shows 400 WBC/mm3, with 51% monocytes, normal glucose, slightly elevated protein, and negative Gram stain for bacteria. Without therapy, she becomes fully alert and afebrile within 24 hours. At this point, she relates that she had two identical episodes within the past 5 years, each after ibuprofen use.

What is the most likely diagnosis for this patient?
A. Recurrent enteroviral aseptic meningitis
B. Systemic lupus erythematosus cerebritis
C. Recurrent herpes simplex aseptic meningitis
D. Ibuprofen hypersensitivity
E. St. Louis encephalitis

Key Concept/Objective: To be aware that both infectious and noninfectious processes can present as an aseptic meningitis or meningoencephalitis syndrome

The patient has evidence of CNS inflammation with a mononuclear CSF pleocytosis. Enteroviruses commonly cause acute viral meningitis, but recurrent acute infections would be very unlikely. St. Louis encephalitis—the other “SLE”—is an arboviral encephalitis that would neither resolve clinically within 24 hours nor recur. Mollaret recurrent meningitis might have a herpesvirus origin, but there is a more likely explanation in this patient. Cerebritis can be part of active systemic lupus erythematosus, but rapid resolution and no other manifestations of active disease argue against the diagnosis. Nonsteroidal anti-inflammatory drugs can cause a meningoencephalitis, especially in patients with underlying collagen vascular diseases. Three identical episodes occurring immediately after the use of ibuprofen, with rapid improvement after its removal, argue for this diagnosis. (Answer: D—Ibuprofen hypersensitivity)

87. Eight weeks ago, a 26-year-old woman with Hodgkin disease developed a zosteriform eruption of the left periorbital and left forehead regions. The skin lesions resolved, and apart from episodic neuralgic pains, the patient was doing well until today, when she experienced the sudden development of right hemiplegia. On examination, the patient is awake, afebrile, and aphasic; residual small scabs are noted on the left side of the face, and she has a dense right hemiplegia.

For this patient, which of the following diagnostic test results is most likely?
A. Increased left frontal and temporal lobe intensity on T2-weighted MRI scan images
B. Isolation of CMV from the buffy coat of the blood
C. Bilateral large and small ischemic infarcts of the white and gray matter on CT scan
D. Mononuclear pleocytosis and presence of HSV-1 antibody in the CSF
E. Beading of the left middle cerebral artery on angiography

Key Concept/Objective: To understand that a vasculopathy (granulomatous arteritis) can occur as a late complication of localized varicella-zoster infection

This patient has just developed a sudden, severe contralateral hemiplegia in relation to the site of the recent localized varicella-zoster ophthalmicus. It is caused by a vasculopathy or granulomatous arteritis, which can occur weeks to months after a local infection. Her presentation is that of a stroke in the left middle cerebral artery distribution. Radiographic scans would show a middle cerebral artery distribution defect rather than the bilateral ischemic white and gray matter infarcts characteristic of herpes zoster encephalitis. Angiography would show segmental changes involving the large arteries, described as beading. Left temporal and frontal lobe involvement on MRI is seen with herpes simplex encephalitis. CMV in the buffy coat indicates active cytomegalovirus in the blood and would not explain the neurologic complication of post–varicella-zoster infection. Mononuclear pleocytosis is likely to be present but not HSV-1 antibody, which is the antibody found in herpes simplex encephalitis. (Answer: E—Beading of the left middle cerebral artery on angiography)

88. A 38-year-old man with AIDS who has been followed in clinic for 5 years is brought in by his partner/caregiver, who has concerns that the patient is showing signs of forgetfulness, inability to concentrate, and loss of ability to perform tasks requiring higher function, such as paying bills and making phone calls. The partner states that he first noticed changes in the patient about a year ago, but the cognitive decline has become more noticeable in the past few months. He has also noticed that the patient has developed a slight tremor and mild unsteadiness of gait. Examination reveals a thin man with oral thrush who has psychomotor slowing and appears apathetic. On neurologic examination, there is a slight intention tremor and difficulty with rapid, alternating movements; other symptoms are nonfocal, and the muscle tone is normal. On the Mini-Mental State Examination, the patient scores 22 out of 30 (when the patient took this examination on his initial visit to the clinic 5 years ago, he scored 29 out of 30). A lumbar puncture is performed; the opening pressure is 11 cm H2O, and the cerebrospinal fluid is normal except for a mildly elevated protein level. Gram stain and India ink smear are negative. Magnetic resonance imaging of the brain shows only cerebral atrophy.

Which of the following is the most likely diagnosis for this patient?

A. Toxoplasmosis
B. HIV dementia
C. Cryptococcal meningitis
D. Alzheimer dementia
E. Parkinson disease

Key Concept/Objective: To be able to recognize the clinical features of HIV dementia

This patient most likely has HIV dementia (HIVD, also called AIDS dementia complex), a disorder of the central nervous system caused by a primary effect of the HIV virus. Before the development of highly active antiretroviral therapy (HAART), HIVD developed in up to 30% of HIV-infected patients. The pathogenesis is not completely understood but is thought to result from the effects of neurotoxins secreted from chronically infected microglia. HIVD generally occurs in the later stages of HIV infection and can present initially as mild cognitive impairment. Presentation is consistent with a subcortical dementia: in addition to loss of memory and language function, patients may demonstrate generalized psychomotor slowing, apathy, ataxia, and even paralysis. The diagnosis depends on the exclusion of other reversible causes of dementia and altered mental status in the patient with AIDS. In this patient, MRI did not reveal ring-enhancing lesions, which would have suggested toxoplasmosis. A normal opening pressure and negative India ink smear, although not completely ruling out cryptococcal meningitis, would certainly make it less likely. The subcortical features in this patient would not be typical of Alzheimer dementia, and the absence of rigidity makes Parkinson disease less likely than HIVD. HAART may improve HIVD. (Answer: B—HIV dementia)

89. A 60-year-old woman is admitted to the hospital from the emergency department because the family is no longer able to care for her at home. The patient has diet-controlled diabetes and had been doing well, but the family now describes mental deterioration, which has been progressing over the past 3 to 4 months. The patient first demonstrated forgetfulness and subsequently developed sleep difficulties, mood swings, and progressively poorer judgment and loss of short-term memory. The family has been struck by the rapidity of the changes in the patient in the past month. At the time of admission, the patient is awake but minimally responsive and has completely lost the ability to perform basic activities of daily living. Examination is significant for frequent myoclonic jerks, which are especially prominent when the patient is startled. She is unable to follow commands, but strength and sensation appear intact. She is markedly ataxic and ambulates only with assistance. The results of CT of the head are normal, as are the results of lumbar puncture.

Which of the following statements regarding the likely diagnosis in this patient is false?

A. The disease is uniformly progressive and fatal
B. Detection of the abnormal 14-3-3 protein in the CSF can help support what is often a difficult diagnosis
C. T2-weighted MRI of the brain may show hyperintensity in the basal ganglia and thalamus
D. In the past, use of cadaveric dural grafts and human pituitary hormones was associated with iatrogenic cases of the disease
E. The majority of cases are either inherited or transfusion-associated

Key Concept/Objective: To understand the clinical features of Creutzfeldt-Jakob Disease (CJD)

This patient has several of the hallmarks of CJD, a rare, transmissible spongiform encephalopathy thought to be caused by the accumulation of an abnormal form of an endogenous protein (prion) in the CNS. There are several recognized forms, including sporadic (which make up the majority of cases), familial, iatrogenic, and variant forms. CJD is uniformly fatal. The most striking finding is a progressive dementia that occurs over weeks to months (as compared to Alzheimer dementia, which progresses over years). Myoclonic jerking, especially with startle, is an important physical finding: its presence in association with dementia of unclear etiology should strongly suggest the possibility of CJD. Other clinical findings may include signs of pyramidal tract involvement, muscle atrophy, cerebellar ataxia, and seizures. There is no gold standard test for diagnosis, but typical findings on MRI of hyperintensity of the basal ganglia and the presence of the abnormal 14-3-3 protein in CSF can support the diagnosis. Diagnostic criteria have been proposed; these include, in addition to the clinical features described, the finding of typical loss of neurons, gliosis, or spongiform degeneration in histopathologic specimens of brain tissue, as well as the demonstration of transmission of neurodegenerative disease from brain specimens to animals. Although cannibalism and the use of cadaveric human tissues such as dural grafts and pituitary hormones pose a risk of spreading the disease, there has been no definitive evidence of spread of prion disease through blood products. (Answer: E—The majority of cases are either inherited or transfusion-associated)

90. A 29-year-old Hispanic woman with HIV presents to the emergency department with gait difficulties and visual disturbances; these symptoms have persisted for several weeks. Her medical history includes previous episodes of oral thrush and Pneumocystis carinii pneumonia, and she is not currently receiving antiretroviral therapy because of problems with compliance. Recently, the patient was found to have a CD4+ T cell count of 75 cells/mm3. On examination, the patient has an appreciable visual-field defect (left homonymous quadrantic defect) and an ataxic gait. After admission to the hospital, an MRI of the brain is performed; this MRI reveals multiple coalescent areas of demyelination in the subcortical white matter of the left occipital lobe and the cerebellum. Results of CSF examination are normal, and toxoplasmosis titers are normal. You suspect that the patient may have progressive PML.

Which of the following statements regarding PML is false?
A. PCR analysis of CSF for the presence of JC virus material has been used to confirm the diagnosis
B. HAART may result in improvement and remission of PML
C. PML is most often a manifestation of early HIV disease
D. PML is an opportunistic infection involving reactivation of latent JC virus in the CNS

Key Concept/Objective: To understand the manifestations and diagnosis of PML

PML is a disease of white matter seen primarily in immunosuppressed patients; it characteristically follows a protracted course. PML is associated with multifocal neurologic defects, which may include motor deterioration, loss of vision, incontinence, aphasia, and sensory defects. Mental-status changes may or may not be present. The disease is caused by reactivation of latent human papillomavirus JC in the CNS and is thus an opportunistic infection. The disease is typically a late manifestation of HIV disease; it occurs in 1% to 5% of HIV-infected persons. The diagnosis should be considered in any person with immunodeficiency who presents with a subacute, progressive illness involving motor function and cognition. Other considerations in the differential diagnosis for this patient include toxoplasmosis infection, CNS lymphoma, and other CNS infections, such as tuberculosis and neurocysticercosis. MRI findings of multiple, nonenhancing white matter lesions that tend to coalesce are typical. These lesions have a predilection for the occipital and parietal lobes. The definitive diagnosis has previously been established only by brain biopsy, but PCR techniques for the detection of JC virus DNA material in the CSF have been used with increasing success (the reported sensitivity and specificity of these techniques are greater than 90%). Improved outcomes in AIDS patients with PML have been reported with the use of HAART. (Answer: C—PML is most often a manifestation of early HIV disease)

91. A woman comes to your clinic with concerns about mad cow disease (bovine spongiform encephalopathy [BSE]). She has heard media reports of infected cattle in other countries and of the possible risk of a similar disease spreading to people. She wants to know if it is safe for her and her children to consume beef and dairy products in the United States.

Which of the following statements about new-variant Creutzfeldt-Jakob disease (nvCJD) is false?

A. Compared to CJD, nvCJD typically develops in younger adults
B. The agent of disease is thought to be a protein, or prion, which is spread by the consumption of animal protein products
C. There have been documented cases in the United States
D. There is currently minimal risk of acquiring the disease from consumption of milk or beef in the United States

Key Concept/Objective: To understand the epidemiology of BSE and its association with nvCJD

BSE is a spongiform encephalopathy that has occurred primarily in the United Kingdom and is associated with the consumption (by cows) of protein supplements derived from ruminant tissue. There have been substantial efforts to reduce the incidence of BSE by banning the feeding of such ruminant-derived tissue to cattle and by the disposal of potentially infected herds. There is evidence that nvCJD may represent bovine-to-human spread of BSE in the United Kingdom. nvCJD typically affects younger adults (mean age at onset, 29 years), causing the typical rapidly progressive cognitive decline seen with CJD. Unlike the sporadic or familial forms of CJD, patients with nvCJD have shown prominent sensory disturbances and psychiatric symptoms. Current epidemiologic evidence suggests that the United Kingdom epidemic will be less widespread than initially thought. There have been no documented cases of BSE or nvCJD in the United States. Thus, the patient can be reassured that dairy products and beef are in general safe to consume. (Answer: C—There have been documented cases in the United States)

92. During a regular office visit, a clinic patient raises concerns about her 12-year-old son’s receiving a booster dose of the mumps-measles-rubella (MMR) vaccine. She read on the Internet about a variant form of measles that can result in an incurable degenerative neurologic disease. She is strongly considering not allowing him to receive the booster, and she wants your opinion. You inform her that the disease she is describing, subacute sclerosing panencephalitis (SSPE), is a rare condition that develops years after exposure to measles.

Which of the following statements would you include in your discussion with this patient?
A. SSPE is no longer a concern because the MMR vaccine that is currently used is a killed-virus vaccine, not a live-virus vaccine
B. You agree that she should not allow her son to complete his MMR series because the risk of developing SSPE is greater than the threat of measles or mumps
C. The rate of development of SSPE is 10 times less after vaccination than after having measles infection
D. SSPE typically develops 7 to 10 years after measles and occurs in about 1 in 500 people infected with measles

Key Concept/Objective: To be able to recognize SSPE, a rare but deadly complication of measles infection

SSPE is a rare sequela of measles infection, occurring in approximately 1 in 100,000 persons infected with measles virus. The disease is characterized by CNS deterioration, which progresses from personality changes and lethargy to myoclonus, dementia, decorticate rigidity, and death. The typical patient with SSPE is younger than 20 years and develops the disease 7 to 10 years after infection with measles. The risk of developing SSPE is at least ten times less after vaccination with live virus than it is after contracting wild-type measles infection. The MMR vaccine is a live-virus vaccine, but given the very small risk of serious sequelae compared to the risk of the adverse effects of actual measles or mumps infection, completion of the immunization series should be advised. (Answer: C—The rate of development of SSPE is 10 times less after vaccination than after having measles infection)

For more information, see González-Scarano F: 11 Neurology: XVII Central Nervous System Diseases Due to Slow Viruses and Prions. ACP Medicine Online (www.acpmedicine.com). Dale DC, Federman DD, Eds. WebMD Inc., New York, June 2002